MAT2345SUMMARY

Download as pdf or txt
Download as pdf or txt
You are on page 1of 112

1

MAT2345: ANALYTIC GEOMETRY

LECTURE NOTES SUMMARY

By Theoneste Hakizimana
2

Unit1: AFFINE EUCLIDEAN SPACES

Def1.1. A real affine space is a triple    A, V ,   , where, A is a set whose elements are called

points, V is a real vector space of which elements are called vectors, and  is a map

 : A  A  V ;( P, Q)   ( P, Q )  PQ, satisfying the following conditions:

P  A, u  V !Q  A : PQ  u (1.1)

P, Q, T  A : PQ  QT  PT (1.2)

If in (1.2) we set P  Q  T , then PP  0 ; which is a zero vector and if in (1.2) we put P  T ,

then QP   PQ , which is the opposite vector to PQ .

The dimension of the affine space A is the dimension of its associated vector space V .

Def1.2. A frame or a coordinate system in An is a pair F  O; B , where O  A and

B  e1 ,..., en  is a basis of V . The point O is called the origin of the coordinate system.

Each coordinate system F  O, e1 ,..., en  of an affine space defines an isomorphism

f : A  Rn : f ( P)  ( x1,..., x n ). (1.3)

Def1.3. The numbers ( x1 ,..., x n ) defined by (1.3) are called affine coordinates of the point P
with respect to the coordinate system F . The coordinates of a point P are the coordinates of

its radius vector OP referred to the basis B i.e. OP  x1e1  ...  x nen

Def1.4. A Euclidean space is a real vector space V equipped with a positive definite symmetric
bilinear form denoted by , . This means that,

, : V  V  , ( x , y ) x , y , x, y, z V ,   :

(i ) x, x  0, ( x, x  0  x  0) (ii ) x , y  y, x

By Theoneste Hakizimana
3

(iii ) x  y, z  x , z  y, z ;  x , z   x , y ; x , y  z  x , y  x , z ; x ,  y   x , y

The Euclidian space will be denoted by V , ,  . The Euclidian structure enables us to introduce

metric notions like length, distance, angle, area, volume, etc.

Def1.5. The value of the bilinear form on the vectors x and y is denoted x , y and is called the

scalar product of the two vectors.

Def1.6. The square root of x , x is called the norm or magnitude or length of the vector x and is

denoted x : x, x (1.4)

x V ,   : (i ) x  0 x  0  x  0 (ii ) x   x

In Euclidean space, two important inequalities hold: x, y, z V ,

(iii ) x, y  x y (iv) x  y  x  y

The inequality (iii) is called Cauchy-Bouniakovsky-Schwarz inequality and the inequality (iv) is
the triangle inequality or Minkowski inequality.

Def1.7. The angle between two non-zero vectors x and y is a real number    0,   defined by

x, y
cos   (1.5)
x y

Def1.8. Two vectors x and y are orthogonal or perpendicular and we write x  y, if and only if

x , y  0 . In Euclidean space, The Pythagorean’s theorem holds

x y xy  x  y
2 2 2
(1.6)

Def1.9. A basis e1 ,..., en  in V is orthogonal if and only if: i, j; i  j  ei , e j  0 (1.7)

By Theoneste Hakizimana
4

1, i  j
A basis e1 ,..., en  in V is orthonormal if i, j , ei , e j   ij   (1.8)
0, i  j

Def1.10. An affine Euclidean space is an affine space such that it is associated vector space is a
Euclidean space.An orthonormal coordinate system in an affine Euclidean space An is a
coordinate system containing an orthonormal basis. It is also called Cartesian system or
rectangular coordinate system.

n n
The expression of the scalar product of vectors x   xi ei and y   y j e j in orthonormal basis
i 1 i 1

n
is given by: x , y   xi y i (1.9)
i 1

 x 
n 2
The magnitude of the vector x in orthonormal basis is given by : x  i
(1.10)
i 1

x y i i

The angle  between x and y is given by : cos   i 1


(1.11)
 x   y 
n 2 n 2
i i

i 1 i 1

1.2. PRODUCTS OF VECTORS IN 2D AND 3D- EUCLIDEAN SPACES

1.1. Scalar product

 
Let   An ,V , , , n  2,3 be a two or a three-dimensional affine Euclidean space.

Def 1.11.The scalar product or dot product of two non-zero vectors is a number equal to the
product of the lengths of these vectors by the cosine of the angle between them.

u , v   u v cos 

If u  v , then   0 , we have, u , u  u or equivalently u 


2
u, u

Def 1.12 The scalar projection of the vector v onto vector u is a scalar defined as

By Theoneste Hakizimana
5

pru v  v cos 

u , v  u pru v  v prv u

The scalar product satisfies the following properties: u , v  ( An , .,. , n  2,3;   :

(i) u , v   v , u  (ii) u , v    u , v  (iii) u  v , w  u, w  v , w

From the commutative and distributive laws, the following identities are satisfied:

u v  u  v  2 u, v u v  u  v  2 u, v
2 2 2 2 2 2

u v  u v
2 2

2 u  v
2 2
 Parallelogram Rule

u , v ; u , v  u v Cauchy-Bouniakovski Inequality

u v  u  v
Triangle inequality

If u and v are perpendicular i.e. u  v  u , v  0 , then

u v  u  v
2 2 2
Pythagorean Theorem

The analytical expressions of the dot product and the angle between two vectors in 2D and 3D
Euclidean spaces equipped with orthonormal bases are given by

In 2
, u  x1e1  y1e2 ; v  x2 e1  y2 e2 :

u , v  x1 x2  y1 y2
u , v   x1 x2  y1 y2 , u  x12  y12 , cos   
u v x  y12 x2 2  y2 2
1
2

In 3
,u  x1e1  y1e2  z1e3 ; v  x2e1  y2e2  z3e3 :

u , v  x1 x2  y1 y2  z1 z2
u , v   x1 x2  y1 y2  z1 z2 ; u  x12  y12  z12 ; cos   
u v x  y12  z12 x2 2  y2 2  z2 2
1
2

By Theoneste Hakizimana
6

1.2. Cross product

Def 1.17. The cross product or the vector product of the vectors u and v is a new vector of V
denoted by [ u , v ] or u  v , satisfying the following conditions:

(1) u , v  is orthogonal to both vectors u and v

(2) u , v   u v sin  ; where  is the angle between vectors u and v

(3) The vectors u , v and u , v  taken in the indicated order, form a right-handed system.

The geometrical significance of the cross product is that the length of the vector u , v  is equal

to the area of the parallelogram with sides determined by u and v . We have

S  u , v   u v sin 

u, u u, v
u , v 
2
 u v  u, v 
2 2 2

v,u v, v Lagrange Identity

u, u u, v
The area S of the parallelogram is given by: S 
v, u v, v

1 1
In particular, the area of triangle with sides u and v is given by: S  u , v   u v sin 
2 2
The cross product satisfies the following properties: u , v , w V ,   R

(i) u , v     v , u  (ii)   u , v   u ,  v    u , v 

(iii)  u , v  w   u , v    u , w  ; u  v , w   u , w    v , w 

(iv) u , v  , w   v , w , u    v , w , u   0 Jacobi Identity

By Theoneste Hakizimana
7

(v) u , v  , w  u , w v  v , w u

The cross products of the basis unit vectors e1 , e2 , e3  are

e1 , e2   e3 : e2 , e3   e1; e3 , e1   e2 ; e1 , e1   e2 , e2   e3 , e3   0

In orthonormal basis, the expression of the cross product of the vectors

u  x1e1  y1e2  z1e3 and v  x2e 1  y2e2  z2e3 is given by:

e1 e2 e3
u , v   ( y1 z2  z1 y2 )e1  ( z1 x2  x1 z2 )e2  ( x1 y2  y1 x2 )e3  x1 y1 z1
x2 y2 z2

1.3. Triple Scalar product or Box Product

Def1.16. The triple scalar product or box product or mixed product of vectors u , v and w is a

scalar quantity denoted u , v , w and defined by: u , v , w  u , v  , w .

The absolute value of the box product of three non-coplanar vectors u , v and w is the volume

of the parallelepiped spanned by these vectors i.e. V  u , v , w

In an orthonormal basis B  e1 , e2 , e3  , the box product of the vectors u  x1e1  y1e2  z1e3 ,

v  x2e1  y2e2  z2e3 and w  x3e1  y3e2  z3e3 is given by:

x1 y1 z1
u , v , w  det  u , v , w   x 2 y2 z2
x3 y3 z3

By Theoneste Hakizimana
8

The scalar product and box product are connected by the following formula

u , u u , v u , w
u , v , w u , v, w  v , u  v , v v , w
w, u  w, v w, w

u, u u, v u, w u, u u, v u, w
u, v, w  v, u
2
v, v v, w V v,u v, v v, w
w, u w, v w, w w, u w, v w, w

Worked Example.

Given the tetrahedron with vertices: A(1,2,0); B(2,1,3); C(1,0,1); D(3, 2,3) .Find

a. the perimeter P of the face ABC b. the angle A of the face ABC

c. the scalar projection of the vector AB on the vector AC

d. the area S of the face ABC . e. Find the volume of the tetrahedron

3  1  2 4


       
Solution:a. AB   1 ; BC   1 ; AC   2 ; AD   4 ; AB  19; BC  6; AC  3 ;
 3  2   1   3 

The perimeter of the triangle is given by: P  AB  AC  BC  19  3  6

 AB, AC  5
b. The cosine of the angle A of the face ABC of is given by: cos A  
AB AC 3 19

AB, AC 5
c. The scalar projection of the vector AB on the vector AC equals:  AC ( AB)  
AC 3

1 1 1
d. The area of the face ABC is : S ABC   AB, AC  =   7    9   42 
2 2
  146
2 2 2
3 1 3
1 1 2
e.The volume of the tetrahedron is given by : V  AB, AC , AD  2 2 1 
6 6 3
4 4 3

By Theoneste Hakizimana
9

UNIT2: TRANSFORMATIONS OF COORDINATES IN AFFINE EUCLIDEAN SPACE

2.1. TRANSFORMATIONS OF COORDINATES

1.Transformation of coordinates in a vector space

Let ( x1 ,..., x n ) and ( x1 ,..., xn ) be the coordinates of the vector v referred to the bases

B  e1 ,..., en  and B  e 1 ,..., en  of the vector space V respectively:

e  e  P (2.1)

P   Pii  is the transition matrix from B to B ,  e    e1...en  and  e   e1 ...en 

If P is the transition matrix from B   e  to B   e and Q is he transition matrix from

B   e to B   e , then PQ is the transition matrix from  e  to  e

n n
Let v   xi ei =  xiei . If X   x1 ... x n  , X    x1 ... x n  then:
T   T

i 1 i 1

X  PX  (2.2)

2. Transformations of coordinates in affine spaces

Let F  O; ei  and F   O; ei  be two coordinate systems in An , P the transition matrix from

e  to e and (a1 ,..., an ) the coordinates of the origin O with respect to the basis  e  i.e.
n n n
OO   a i ei . If OM   x i ei and OM   x iei ,then X  PX   a
i 1 i 1 i 1

 p11 ... p1n a1 


 
Let  P, a   ... ... ... ...  , be the transition matrix from F  O; e  to F   O; e
 p1n ... pnn a n 
 

By Theoneste Hakizimana
10

If  P, a  is the transition matrix from F to F  and Q, b  is the transition matrix from F  to

F   O; e1,..., en , then the transition matrix from F to F  is given by :  PQ, Pb  a 

Example:In the affine plane, find the coordinates of the point M  x, y  with respect to the old

 1   1 
coordinate system F  O  2, 1 ; e1    , e2     if its coordinates in the new coordinate
 5   4 

 1  4 
system F   0  3, 2  ; e1    ; e2     are M  x, y 
  1  2 

 x   a1   x 
Solution: The coordinates of the point M x, y  are given by :       P   ,
 y   a2   y

where a1 , a2  are the coordinates of the new origin O with respect to

the basis e1 , e 2  i.e. OO  a1e1  a2e2 and P is the transition matrix
 

 3  2  1
Since OO          , the coordinates (a1 , a2 ) of the vector OO are obtained
 2  1 3

1 1  1


 a1 , a2   
7 2
by solving,    a1 5  a2  4  leading to , 
 3     9 9
To find the transition matrix P , we should solve the following pairs of systems of equation.
1 1  1 4 1   1
 1  p11    p21  ; 2  p12    p22  
  5  4   5 4
 p11  p21  1  p12  p22  4
 ; 
5 p11  4 p21  1 5 p12  4 p22  2
 1 
2
p p12   3
Solving, we obtain: P   11  .
 p21 p22   2
 2 
 3 
1 7 2 2
The coordinates of M are: x  x  2 y   ; y   x  2 y  
3 9 3 9

By Theoneste Hakizimana
11

Example2. In the affine 3D space, find the coordinates of the point M  x, y, z  with respect to

  3  3 1  
 
the old coordinate system, F  O 1,3,3 ; e1  3 , e2   5  , e3   2   if its coordinates in the
   
 1  2   1  

 1  4  2 
 
new coordinate system F   O  1, 0, 2  ; e1  2  , e2   2  , e3   1  are M x, y , z 
   
  1  1   3  

 1 1  2


Solution: OO   0   3   3
 2  3  1

  2 3 3  1 
OO  a1e1  a2e2  a3e3 , corresponds to the system:  3  a1 3  a 2 5  a3 2 ,
     
       
 1  1  2 1 

3a1  3a2  a3  2  a1  0 
  a   1 
which can also be written as : 3a1  5a2  2a3  3 , Solving, we get:  2  
a  2a  a  1  a3   1
 1 2 3

 p11 p12 p13 


 
For finding the transition matrix P , we must solve:  e1 e2 e3    e1 e2 e3   p 21 p 22 p 23 
 p 31 p32 p33 

This is equivalent to
1  3 3  1  4 3 3  1 
 2  p 3  p 5   p  2  2  p 3  p 5   p  2  ;
  11   21   31     12   22   32  

1  1   2 1  1  1   2  1 


;
2  3 3  1 
 1  p 3  p 5   2
  13   23    
3  1  2 1 
Solving the systems of equations and using the formula:
 x   a1   x   x  4 x  3 y   6 z 
 y    a   P  y , we obtain: 
   2    y  8 x  3 y  13z  11
 z   a3   z    y  13x  4 y  3z   1

By Theoneste Hakizimana
12

2.2. TRANSFORMATIONS OF COORDINATES IN 2D AND 3D EUCLIDEAN SPACES

1. Translation and Rotation in affine Euclidean plane

 Let F  O; e1 , e2  and F   O; e1 , e2  be orthonormal frames    A2 , ,  , such that

e1  e1 ; e2  e2 ; OO  x0 e1  y0 e2 . If ( x, y) and ( x, y) are the coordinates of the point M

w.r.t F and F  , then the following translation formulas are satisfied

 x   x   x0 
 y    y   y  0r x  x  x0 ; y  y  y0
     0

 Let F  O; e1 , e2  and F   O; e1 , e2  be orthonormal frame in Euclidean    A2 , ,  ,

such that O  O;  e1 , e1    e2 , e2    . If ( x, y) and ( x, y) are the coordinates of

the point M w.r.t F and F  , then the following rotation formulas are satisfied

 x  cos   sin    x 
 y    sin  cos    y
0r x  x cos  y sin  ; y  x sin   y cos
  

 Let F  O; e1 , e2  and F   O; e1 , e2  be orthonormal frames    A2 , ,  , such that

OO  x0e1  y0e2 ;  e1 , e1    e2 , e2    . If ( x, y) and ( x, y) are the coordinates of

the point M referred to the coordinate systems F and F  respectively, then the
analytic expression of a rotation followed by a translation in affine Euclidean plane
are governed by the following formulas

 x  cos   sin  x0   x 
 x  cos   sin    x   x0 
 y    sin        or  y    sin  cos  y0   y
   cos    y  y0 
 1   0 0 1   1 

x  x cos  y sin   x0 ; y  x sin   y cos  y0

By Theoneste Hakizimana
13

Example1:Let the coordinates of the point M in the old system F  O; e1 , e2  be (1, 1) and in
the new system F   O; e1 , e2  be (3, 4) . Find the coordinates ( x0 , y0 ) of the new origin point
O in the old system.
Solution: Since ( x, y)  (1, 1); ( x, y)  (3, 4), O( x0 , y0 ) , using formulas x  x  x0 ; y  y  y0 ,
we have, 1  3  x0 ;  1  4  y0 .Solving, we get O(2,3) .
Example2:Given the equation of hyperbola xy  1 . Find the equation of the same parabola in
the new coordinate system which was obtained from the old system by rotating the axes

anticlockwise through an angle
4

Solution: Since   , using rotation formulas formula
4
x  x cos  y sin   x0 ; y  x sin   y cos  y0 , we have,

2 2
x ( x  y); y  ( x  y) 1  3  x0 ;  1  4  y0 .
2 2

Substituting the values of x and y into the equation xy  1 , we get, x2  y2  2 .

Example3.:If the orthonormal frame F   O; e1 , e2  is obtained from the orthonormal frame


F  O; e1 , e2  via a rotation through the angle followed by a translation generated by a
6
vector whose coordinates referred to the frame F  O; e1 , e2  are (1, 1) , then the coordinates

( x, y) and ( x, y) of the point M referred to the frames F  O; e1 , e2  and F   O; e1 , e2 

respectively are related by

   
x  x cos  y sin  1; y  x sin  y cos  1 or
6 6 6 6
3 1 1 3
x x  y  1; y  x  y  1
2 2 2 2

By Theoneste Hakizimana
14

2. Translation and orthogonal transformations in 3D affine Euclidean space

Prop. Let F  O; e1 , e2 , e3  and F   O; e1 , e2 , e3  be orthonormal coordinate systems in 3D

 
Euclidean affine space   A3 , , , such that e1  e1 ; e2  e2 ; e3  e3 ; OO  x0e1  y0e2  z0e3 .

If ( x, y, z) and ( x, y, z) are the coordinates of the point M referred to the coordinate systems

F and F  respectively, then the two coordinates are related by the translation formulas

 x   x   x0 
 y    y   y 
     0
 z   z   z0 

Pr. Let F  O; e1 , e2 , e3  and F   O; e1 , e2 ; e3  be orthonormal coordinate systems in 3D


Euclidean space   A3 , , , such that 
 p11 p12 p13 
O  O;  e1 e2 e3   e1 e2 e3   p21 p22 p23   e  e  P; PT  P 1 .
 p31 p32 p33 

If ( x, y, z) and ( x, y, z) are the coordinates of the point M referred to the coordinate systems

F and F  respectively, then the two coordinates are related by the following formulas

 x   p11 p12 p13   x 


 y   p p22 p23   y
   21
 z   p31 p32 p33   z  

The formulas of the orthogonal transformations followed by a translation in 3D Euclidean


space are governed by the following equations

 x   p11 p12 p13   x   x0 


 y   p p22 p23   y   y0 
   21
 z   p31 p32 p33   z   z0 

By Theoneste Hakizimana
15

Example: In 3D Euclidean space, let consider the transformation governed by the equations:

 1  3 
 0
2 2
 x    x   1 
 y   3 1 
0   y   2 
   2 2
 z     z    1
 0 0 1    
 
 

Find the coordinates of the point M if in the coordinates system ( x, y, z) if its coordinates in
the system ( x, y, z ) are (1,0, 1) .

Solution: Geometrically, this transformation represents the composition of the rotation of



Oxyz  coordinate system through an angle counterclockwise about z  axis (looking along
3
1
the positive z  axis toward the origin) followed by the translation by the vector v   2  .
 1

The coordinates of the point M if in the coordinates system ( x, y, z) are given by

 1 3 
 0
2 2
 x     x 1   x  1   x    3 
 y    3 1         y     1 
   2 0   y  2  . Since  y    0  ;   , we have    
 2   z 1 4
 z    0    z   1  z    0 
0 1  
 
 

By Theoneste Hakizimana
16

2.3. POLAR COORDINATES AND ITS GENERALISATIONS

2.3.1. Polar coordinates

  OM : polar radius or radial coordinate;   Ox, OM  . polar angle or angular


coordinate. From the Fig, the Cartesian coordinates ( x, y) and the polar coordinate (  ,  ) of
the point M are related by:

x y
x   cos ; y   sin  ;   x 2  y 2 ; cos   ; sin  
x2  y 2 x2  y 2

Example1: Find the polar coordinates of the point M if its Cartesian coordinates are M (1,1) .
1 1
Solution: Using formula (1.52), we have   (1) 2  12  2; cos   ;sin   .
2 2
3 3
The polar angle is   .The polar coordinates of the point M are M ( 2, ).
4 4

Example2.Find the Cartesian coordinates of the point M if its polar coordinates are M (4, ) .
3
 
Solution. we have x  4 cos  2; y  4sin  2 3 .Thus, M (2, 2 3)
3 3
Example3: Transform the equation   2sin  in Cartesian rectangular form and then
determine the type of curve it represents.
Solution: Multiplying both sides of the equation   2sin  by  , we have  2  2  sin 

Substituting  2  x2  y 2 and  sin   y in the above equation, we have: x2  y 2  2 y

The completion of square in y leads to x 2   y  1  1 .Unit circle with center at C (0,1) .


2

By Theoneste Hakizimana
17

2.3.2 Cylindrical co-ordinates

M  ,  , z 

The cylindrical coordinates of a point M of space are represented by an ordered triple   ,  , z 

, where (  ,  ) are polar co-ordinates of the orthogonal projection M  of M onto the xy -plane
and z is the usual third rectangular coordinate of M . From the Fig, the Cartesian coordinates

 x, y, z  and cylindrical coordinates   ,  , z  of the point M are connected by:

y
x   cos ; y   sin ; z  z ;   x 2  y 2 ;   arctan ; z  z
x

2
Example1.The cylindrical co-ordinates of the point M are (2, ,1) , determine its Cartesian co-
3
ordinates.

2
Solution. If (2, ,1) are the cylindrical coordinates of M ,, its corresponding Cartesian co-
3
2 2
ordinates are given by x  2 cos  1; y  2sin  3; z  1 ,i.e. M (1, 3,1)
3 3

Example2: Determine the cylindrical coordinates of the point M if its Cartesian coordinates
are (3, -3, -7)

By Theoneste Hakizimana
18

3 7
Solution. We have:   32  (3) 2  3 2;arctan( ) ; z  7 i.e. the cylindrical
3 4
7
coordinates of the point are M (3 2, , 7)
4

2.3.3: Spherical co-ordinates

In spherical coordinates, a point M of space is represented by an ordered triple (  ,  , ) , where  is a


polar angle associated with the projection M  of M onto the xy - plane, is the angle between the

positive z - axis and OM ;   OM . From the figure, the Cartesian coordinates  x, y , z  and the

spherical coordinates   ,  ,  of the point M are related

x   sin cos ; y   sin sin ; z   cos

y z
  x 2  y 2  z 2 ;  arctan ;  ar cos ,   0;0    ,0   2
x x2  y 2  z 2

The name spherical coordinates is used because of the equation of the graph   r is a sphere
of radius r centered at the origin.

Example: a. Determine the Cartesian co-ordinates of the point M if its spherical co-ordinates

 
are (2, , )
4 3

By Theoneste Hakizimana
19

 
b. If the Cartesian co-ordinates of a point M are 0, 2 3,  2 , find its corresponding
spherical coordinates.
  6   6  6 6
Solution: a. x  2sin cos  ; y  2sin sin  ; z  2cos  1 i.e. M ( , ,1)
3 4 2 3 4 2 3 2 2

2 3   1  2   2 
b.   0  12  4  4;   arctan    ;  ar cos    i.e. M  4, , 
 0  2  2  3  2 3 
 
Example:

Change the equation   6 sin cos  4 sin sin   8 cos  0 to rectangular coordinates

Solution: This equation is expressed in spherical coordinates. Multiply trough by  , we have

 2  6 sin cos   4 sin sin   8 cos  0 , or x 2  y 2  z 2  6 x  4 y  8z  0


Completion of square gives:  x  3   y  2    z  4   29
2 2 2

This is the equation of sphere with its center at C  3, 2, 4  and radius r  29

Example: Change the equation:   2cos to rectangular coordinates

Solution: Multiplying each side of equation by  gives  2  2 cos ,

then substitution of  2  x2  y 2  z 2 ; z   cos yields


x2  y 2  z 2  2z
Completion of square in z gives

x 2  y 2   z  1  1
2

This equation represents a unit sphere centered at the point C  0, 0,1

By Theoneste Hakizimana
20

Unit3: ELEMENTARY PROBLEMS OF PLANE GEOMETRY

3.2. ELEMENTARY PROBLEMS OF EUCLIDEAN PLANE

 
Let   A2 , V , , be an affine Euclidean plane endowed with orthonormal frame F (O; e1 , e2 ) .

1.1. 3.2.1. Distance between two points

The distance d between the points P( x1 , y1 ) and Q( x2 , y2 ) is given by

d ( P, Q)  ( x2  x1 ) 2  ( y2  y1 ) 2
(3.1)

In polar coordinates, the distance d between the points P ( 1 , 1 ) and Q(  2 ,  2 ) is given by

d ( P, Q)  12   22  2 1  2 cos(2  1 ) (3.2)

Example1: Determine the distance between the points P(3,1) and Q(1, 4)

Solution: Using formula (3.1), d ( P, Q)  (1  3) 2  (4  1) 2  5

Example2: Determine the distance between P and Q , if their polar coordinates are

    
P 5,  and Q 8, 
 4  12 

Solution. Applying the formula (3.2), d P, Q   12   22  2 1  2 cos 2  1  :

  
d P, Q   5 2  8 2  2  5  8 cos    7
 4 12 

The distance satisfies the following properties: P1 , P2 , P3  (d ) :

(i) d ( P1 , P2 )  0 and d ( P1 , P2 )  0  P1  P2 ) ; (ii) d ( P1 , P2 )  d ( P2 , P1 )

(iii) d ( P1 , P3 )  d ( P1 , P2 )  d ( P2 , P3 )

By Theoneste Hakizimana
21

1.2. 3.2.2. Division of a line segment in given ratio

Recall that the point M divides the line segment PQ in the ratio  if and only if PM   MQ.

If P( x1 , y1 ) and Q( x2 , y2 ) , then the co-ordinates ( x, y) of the point M dividing the segment

PQ in the ratio  are

x1   x2 y1   y2
x ; y
1  1  (3.3)

Note.

If   0, M is inside of the segment PQ , if   0 , M is outside of the segment PQ

If   0 , then M  P ; If    , then M  Q

If   1 , the point M moves away indefinitely on the line carrying PQ

If M is the midpoint of PQ , then   1 and therefore we obtain the midpoint formula

x1  x2 y y
x ;y 1 2 (3.4)
2 2

The coordinates of the centroid G ( x, y ) of a system of material points Pi ( xi , yi ) with masses

mi (i  1,..., n) are given by

m1 x1  ...  mn xn m y  ...  mn yn
x ;y 1 1
m1  ...  mn m1  ...  mn (3.5)

Example. Let A( x1 , y1 ), B( x2 , y2 ) and C ( x3 , y3 ) be the vertices of the triangle ABC . Find the co-

ordinates of the centroid G of the triangle ABC .

Solution. The centroid G of the triangle ABC (intersection of medians) is given by

 x  x  x y  y2  y3 
G 1 2 3 , 1 . (3.6)
 3 3 

By Theoneste Hakizimana
22

1.3. 3.2.3. Area of a triangle

Let A( x1 , y1 ), B( x2 , y2 ), C ( x3 , y3 ) be vertices of the triangle ABC .

The area S ABC of the triangle is governed by the formula

1
S ABC   x1  x2  y1  y2    x2  x3  y2  y3    x3  x1  y3  y1   (3.7)
2

In terms of determinants the area of the triangle is written

x1 y1 1
1 1 x2  x1 y2  y1
S ABC   x2 y2 1   (3.8)
2 2 x3  x1 y3  y1
x3 y3 1

In polar coordinates the area of the triangle A(r1 , 1 ), B(r2 , 2 ) and C (r3 , 3 ) is given by

1
S ABC   r1r2 sin 2  1   r2 r3 sin 3  2   r3r1 sin 1  3   (3.9)
2

1
If r3  0 , then S ABC  r1r2 sin 2  1  (3.10)
2

1 1 1
S ABC  ab sin C  bc sin A  ac sin B (3.11)
2 2 2

x1 y1 1
Three points A( x1 , y1 ), B( x2 , y2 ), C ( x3 , y3 ) are collinear if and only if x2 y2 1  0
x3 y3 1

By Theoneste Hakizimana
23

Example1: Find the area of the triangle whose vertices are A(1,1); B(6, 4); C(8, 2)

1 1
Solution. S ABC  (1  6)(1  4)  (6  8)(4  2)  (8  1).(1  2)  16  8
2 2

 5
Example2.Find the area of the triangle whose vertices are A(0, 0); B(4, ); C (1, )
9 18

Solution. Applying the formula (3.9), we have

1    5    5   1 
S ABC   0  4  sin   0   4 1 sin     1 0  sin  0     4sin  1
2 9   18 9   18   2 6

Note: The area of a polygon whose vertices are P1 ( x1 , y1 ), P2 ( x2 , y2 ), , Pn ( xn , yn ) is given by

1
S ( x1  x2 )( y1  y2 )  ( x2  x3 )( y2  y3 )    ( xn  x1 )( yn  y1  (3.12)
2

In polar coordinates, the area of a polygon with vertices P1 ( 1 , 1 ), P2 ( 2 , 2 ), , Pn ( n , n ) is

1
S  r1r2 sin 2  1   r2 r3 sin 3  2   ...  rn 1rn sin n  n 1   rn r1 sin 1  n   (3.13)
2 

Example3.Find the area of the quadrilateral with vertices A(1,1); B(0, 2); C(5,0); D(8, 6)

Solution. Applying the formula (3.12), we have

1
S ABCD  (1  0)(1  2)  (0  5)(0  2)  (5  8)(0  6)  (8  1)(6  1)  25
2

   3       
Example4. Find the area of the quadrilateral A  2,  ; B  2,  ; C  5,  ; D  3, 
 3  4   2  6 

Solution. Applying the formula (3.13), we obtain

1   3     3   5        41  13 3
S ABCD   2 2 sin  4  3   5 2 sin  2  4   15  sin      6sin     
2       6 2  3 6  4

By Theoneste Hakizimana
24

UNIT4: STRAIGHT LINE IN 2D AFFINE EUCLIDEAN SPACE

1.4. 4.1. Various equations of a straight line

Def. A nonzero vector u  0 parallel to a straight line (d ) is called its direction vector.

Suppose the straight line (d ) passes through the point M 0 ( x0 , y0 ) and the vector u be its

direction vector. If M ( x, y) is an arbitrary point on the line (d ) , then the vector M 0 M is

collinear to the vector u i.e. there is a real number  , such that M 0 M  u (4.0)

Let r0  OM 0 and r  OM be the position vectors of the points M 0 and M respectively.

Since OM  OM 0  M 0 M , the equation (4.0) may be written

(d ) : r  r0  u , (4.1) : vector equations

If (u 1 , u2 ) are the co-ordinates of the vector u , then the equation (4.1) in co-ordinates form is

 x  x  u 
(d ) :     0     1  (4.2) : parametric equations
 y   y0  u2 

Eliminating the parameter  in (4.2), we get

x  x0 y  y0
(d ) :  (4.3) : canonical equations
u1 u2

From equation (4.3), we have u2 x  u1 y  u2 x0  u1 y0  0

Putting A  u2 , B  u1 , C  u2 x0  u1 y0 , we obtain

(d ) : Ax  By  C  0 (4.4) : Analytic implicit equation

A C
If equation (4.4) , B  0, dividing both sides by B and setting, k   ; m   we obtain
B B

(d ) : y  kx  m (4.5) : analytic explicit equation

By Theoneste Hakizimana
25

The equation of the line (d ) crossing Ox; Oy axes at the points A(a,0); B(0, b) is given by:

x y
(d ) :  1 (4.6) : intercept form equation
a b

The general equation Ax  By  C  0 of a line is said normalized if A2  B2  1.

Let Ax  By  C  0 be the general equation of the line (d ) . Dividing both sides of equation by

 A2  B 2  0, where the sign of the radical is chosen opposite to the sign of C when C  0
A B C
and the same as the sign of B when C  0 , we have x y  0,
A B
2 2
A B
2 2
A  B2
2

A B C
Denoting, cos   ;sin   ;p , where  is the angle between
A2  B 2 A2  B 2 A2  B 2
x  axis and the normal vector n , the normal equation of the line is written

(d ) : x cos   y sin   p  0 (4.7) : Normal equation

Note. The absolute value of p is the distance from origin O(0,0) to the line (d ).

Consider a system of polar coordinates   ,   with pole at the point O . Suppose given a

straight line (d ) . Let P be the foot of the perpendicular dropped from the pole to the given line
and ( p,  ) be its polar coordinates (Fig)

Let (  ,  ) be polar coordinates of an arbitrary point M lying on the line (d ).

From the triangle OPM , we have OP  OM cos(   ), or equivalently

p
 (4.8) Polar equation
cos(   )

By Theoneste Hakizimana
26

Note. The polar equation of a straight line can readily be obtained from its normal equation
x cos   y sin   p  0 by means of the substitution x   cos ; y   sin 

If the line passes through the pole, it admits the obvious equation:   

Note: The vector, parametric and Cartesian equations of the line (d ) passing through the

points P( x1 , y1 ); Q( x2 , y2 ) are respectively given by: (d ) : v  P   PQ

x  x  x  x 
(d ) :     1     2 1  (3.9)
 y   y1   y2  y1 

x y 1
x  x1 y  y1
(d ) : x1 y1 1  0
x2  x1 y2  y1
x2 y2 1
4.2. Mutual positions of two straight lines

Two lines (d1 ) : A1 x  B1 y  C1  0 and (d2 ) : A2 x  B2 y  C2  0

A1 B1 C1
 coincide  (d1 )  (d 2 )  if and only if   
A2 B2 C2

A1 B1 C1
 are parallel  (d1 ) ( d 2 )  if and only if   (4.10)
A2 B2 C2

A1 B1
 intersect (d1 )  (d 2 )  P if and only if 
A2 B2

1.5. 4.3. Angle between two straight lines

 If (d1 ) : A1 x  B1 y  C1  0 and (d2 ) : A2 x  B2 y  C2  0 , the angle between them is given

A1 A2  B1 B2
cos   (4.11)
A  B12 . A22  B22
1
2

 (d1 )  (d 2 )  iff A1 A2  B1B2  0


 If (d1 ) : y  k1 x  m1 and (d2 ) : y  k2 x  m2 , then the angle between them is given by

By Theoneste Hakizimana
27

k2  k1
tan   (4.12)
1  k1k2

 (d1 )  (d 2 )  if f k1k2  1 and  (d1 ) ( d 2 )  iff k1  k 2 .

1.6. 4.4. Pencil of straight lines

A pencil of straight lines centered at the point P ( x0 , y0 ) is a set of all straight lines passing

through that point. The point P ( x0 , y0 ) is called the vertex of the pencil.

Let (d1 ) : A1 x  B1 y  C1  0 and (d2 ) : A2 x  B2 y  C2  0 be two non-parallel lines and P0 ( x0 , y0 )

be their intersecting point. The general equation of a line passing through P0 ( x0 , y0 ) is given by

A1 x  B1 y  C1   ( A2 x  b2 y  C2 )  0 (4.13)

If   0 , then we get (d1 ) .

Dividing both sides of (4.13) by  and passing to the limit as   , we get (d 2 ) .

The lines ( d1 ) and ( d 2 ) are fundamental straight lines of pencil centered at the point P0 ( x0 , y0 ) .

Note: Three lines (d1 ) : A1 x  B1 y  C1  0;(d2 ) : A2 x  B2 y  C2  0 and (d3 ) : A3 x  B3 y  C3  0

A1 B1 C!
are concurrent if and only A2 B2 C2  0 (4.14)
A3 B3 C3

1.7. 4.5. Distance from a point to a straight line

Def.The distance from the point M 0 to the line (d ) is the length of the perpendicular M 0 M1 ,

dropped from this point onto the line (d )

The distance from the point M 0 ( x0 , y0 ) to the line (d ) : Ax  By  C  0 is given by

By Theoneste Hakizimana
28

Ax0  By0  C
d ( M 0 , (d ))  (4.15)
A2  B 2

Note: The distance d between two parallel lines (d1 ) : Ax  By  C1  0 and

C2  C1
(d2 ) : Ax  By  C2  0 is given by d 
A2  B 2

4.6. Orthocenter, Circumcenter and Incenter of a triangle

Def. The orthocenter H of a triangle is the point of intersection of its altitudes.

The circumcenter O of a triangle is the point of intersection of the perpendicular bisectors


of the sides of the triangle. It is the center of circumcircle.

The incenter I of a triangle is the point of intersection of the internal bisectors of angles of
a triangle. It is the center of incircle.

The coordinates of the orthocenter H ( x, y), circumcenter O( x, y) and incenter I ( x, y) of a

triangle whose vertices are A( x1 , y1 ), B( x2 , y2 ) and C ( x3 , y3 ) are given respectively by

 y1 x2 x3  y12 1 x12  y2 y3 x1 1 
 
 y2 x3 x1  y2 1 x2  y3 y1
2 2
x2 1 
 y xx y2 1 x2yy x3 1 
H ( x, y )   
3 1 2 3 3 1 2
, (4.16)
 x1 y1 1 x1 y1 1 
 
 2 x2 y2 1 2 x2 y2 1 
 x3 y3 1 x3 y3 1 
 

 x12  y12 y1 1 x1 x12  y12 1 


 2 
 x2  y2 y2 1 x2 x2  y2 1 
2 2 2

 x2y2 y 1 x x2y2 1
O ( x, y )   
3 3 3 3 3 3
, (4.17)
 x1 y1 1 x1 y1 1 
 
 2 x2 y2 1 2 x2 y2 1 
 x3 y3 1 x3 y3 1 
 

By Theoneste Hakizimana
29

 ax  bx2  cx3 ay1  by2  cy3 


I ( x, y )   1
a  b  c 
, (4.18)
 abc

a, b, c are respectively the lengths of the sides opposite to the angles A, B, C of the triangle

The area of a triangle formed by the three lines A1 x  B1 y  C1  0; A2 x  B2 y  C2  0 and


2
A1 B1 C1
A2 B2 C2
A3 B3 C3
A3 x  B3 y  C3  0 is given by: S  (4.19)
A1 B1 A2 B2 A3 B3
2
A2 B2 A3 B3 A1 B1

Example.Given the triangle whose vertices are A(4,0) ; B(4,0) and C (0, 4)

a. Find the different types of equations for the sides AC and BC

b. Find the equation of the altitude dropped from the vertex B

c. Find the length of the altitude dropped from B

d. Calculate the angles of the triangle

e. Find the orthocenter, circumcenter and in center of the triangle

f. Find the area of the triangle via the equations of its sides

Solution:

a. Various equations of the side AC

 4 
Since AC    , the vector equation of AC is AC : v  A   AC
 4

 x    4  4
The parametric equations of AC are AC :         
 y  0   4

x4 y
The symmetric or canonical equations of AC are AC : 
4 4

By Theoneste Hakizimana
30

The Cartesian equation of AC is AC : x  y  4  0

1 1 4
The normal equation of AC is AC : x y  0 or
2 2 2
 7   7 
AC :  Cos  x   sin y  2 2  0
 4   4 

 7 
The polar equation of AC is AC :   2 2 sec   
 4 

(i) Various equations of the side BC

 4 
Since, BC    , the vector equation of BC is BC : v  B   BC
4

 x  0  4
The parametric equations of BC are BC :         
 y   4 4

x y4
The canonical or symmetric equations of BC are BC : 
4 4

The Cartesian equation of BC is BC : x  y  4  0

x 1
The normal equation of B is BC :  y2 2 0
2 2

The polar equation of BC is BC :   2 2 sec     


4 

b. The altitude dropped from B is perpendicular to the side AC i.e. hB  AC

Since AC  y  x  4 , the equations of the height hB are given by hB  y   x  k

To find k, we should use the fact that B  hB . We have, 0  4  k  k  4

Therefore, the equation of the altitude dropped from B is hB  y   x  4

c. The length of hB is the distance from the vertex B to the side AC

By Theoneste Hakizimana
31

Ax0  By0  C 404


d  P, ( d )   , we have, hB  d B, AC   4 2
A2  B 2 12   1
2

d. Angles of the triangle: Since CA : y  x  4 and CB : y   x  4 are perpendicular i.e.

CA  CB , then Cˆ  CA, CB   2 . Since AC  BC  4 2 ,the triangle is


isosceles and therefore the remaining angles are A  B 
4

e. Since, a  BC  4 2; b  AC  4 2; c  AB  8 , applying formulas (2.31),(2.32)

and(2.33) to the triangle A(4,0), B(4,0) and C (0, 4) , the coordinates of the
orthocentre, circumcentre and in centre of the triangle are respectively

 0 0 1 16 4 1 
 
 0 0 1 16 4 1 
 4 0 1 0 0 1 
H ( x, y )   ,   H (0, 4)
 4 0 1 4 0 1 
2 4 0 1 2 4 0 1
 
 0 4 1 0 4 1 

 16 0 1 4 16 1 
 
 16 0 1 4 16 1 
 16 4 1 0 16 1 
O ( x, y )   ,   O (0, 0)
 4 0 1 4 0 1 
2 4 0 1 2 4 0 1 
 
 0 4 1 0 4 1 
 

 4 2  (4)  4 2  (4)  8  0 4 2  (0)  4 2  (0)  8  4   4 


I ( x, y )   ,   I  0, 
 4 2  4 2 8 4 2  4 2 8   1 2 

f. Writing the equations of the three sides in the form:


AB : y  0; BC : x  y  4  0; AC :  x  y  4  0

and applying formulas (4.19) the area of the triangle is


2
0 1 0
1 1 4
1 1 4
S  16
0 1 1 1 1 1
2
1 1 1 1 0 1

By Theoneste Hakizimana
32

UNIT5: SECOND ORDER CURVES IN STANDARD FORM

5.1. THE CIRCLE

Def. A circle is the locus of points in a plane whose distance from some fixed point is constant.
The given distance is called the radius and the fixed point is called the center.

The equation of the circle  with radius a and center at point C ( x0 , y0 ) is given by

 : ( x  x0 ) 2  ( y  y0 ) 2  r 2 (5.1)

Expanding, this equation can be written in the form

 : x2  y 2  2mx  2ny  p  0 (5.2)

Where, m   x0 , n   y0 , p  x0 2  y0 2  r 2

If the center is at C( x0 , y0 )  (0,0) , then:  : x2  y 2  r 2 (5.3)

The equation of the circle that passes through 3 points A( x1 , y1 ), B ( x2 , y2 ) and C ( x3 , y3 ) is:

x2  y 2 x y 1
x 2  y12 x1 y1 1
 : 12 0 (5.4): 3-point form equation
x2  y2 2 x2 y2 1
x32  y32 x3 y3 1

The parametric equations of the circle with center at C ( x0 , y0 ) and radius r are

 x  x0  r cos t
 (5.5)
 y  y0  r sin t

By Theoneste Hakizimana
33

The polar equations of the circle of radius r with center at C  0 , 0  is

 :  2  02  2 0 cos   0   r 2 (5.6)

Where,   ,   are the polar co-ordinates of an arbitrary point of the circle.

Example. The sides of a triangle ABC are described by the following equations

AB : x  y  8; AC : 2 x  y  14; BC : 3x  y  22

a. Determine the Cartesian equations of the circumcircle

b. Write the parametric equations of the circumcircle

Solution: a. The coordinates of the vertices of the triangle ABC are determined by solving:

x  y  8 x  y  8 3x  y  22
A:  ; B: ; C:  A  6, 2  ; B  7,1 ; C  8, 2 
2 x  y  14 3x  y  22 2 x  y  14

Writing the equation of circumcircle in the form x 2  y 2  2mx  2ny  p  0 , and substituting
the coordinates of the vertices A, B, C in the above equation, we have

12m  4n  p  40

14m  2n  p  50
16m  4n  p  68

Solving the system, yields m  3, n  2 : p  12

Thus, the equation of the circumcircle is :  : x 2  y 2  6 x  4 y  12  0

We can obtain the same result using three-point form equation (3.4)

x2  y 2 x y 1
40 6 2 1
: 0
50 7 1 1
68 8 2 1

 : x 2  y 2  6 x  4 y  12  0

By Theoneste Hakizimana
34

By completion of squares, we have  :  x  3   y  2   5 2


2 2

The circumcenter and circumradius are O  3, 2  ; r  5

b. The parametric equations of the circumcircle are x  3  5cos t ; y  2  5sin t

Intersection of a circle and a line

To determine the points of intersection of a line (d ) : x  x0  v1; y  y0  v2 (5.7) with the

circle(5.2), substitute the values of x, y from (5.7) in (5.2) and arrange in powers of  . :

a 2  2b  c  0
(5.8)

Where, a  v12  v22 ; b   x0  m  v1   y0  n  v2 ; c  x02  y02  2mx0  2ny0  p

The roots in  of the quadratic equation (5.8) are the distances from the point M  x0 , y0  on the

line (5.7) to the points in which this line intersects the circle. Three cases can occur:

(i) The equation admits a single solution. In this case, the line is tangent to the circle
and the single solution is the point of tangency.

(ii) The equation has no solution. In this case, the line passes out of the circle, the line
and the circle do not intersect.

(iii) The equation has two real distinct roots. In this case, the line meets the circle in
two points. It is called the secant.

Example. Find the coordinates of the points of intersection of the line (d ) : x  2  ; y  1  

and the circle  : x2  y 2  2 x  4 y  1  0

Solution. Substituting x  2  ; y  1   into the equation of the circle, after simplification we

get,  2 1  0. Solving, we obtain two real roots 1  1 and 2  1 .

The line intersects the circle in two distinct points. Substituting the value of obtained roots into
the equation of the line, the intersecting points are M 1 (3, 2) and M 2 (1, 0) respectively.

By Theoneste Hakizimana
35

 The equation of the tangent line to the circle  : x2  y 2  2mx  2ny  p  0 at the point

T ( x0 , y0 ) is given by x0 x  y0 y  m( x0  x)  n( y0  y)  p  0

In particular, the equation of the tangent line to the circle  : x2  y 2  r 2 at the point

T ( x0 , y0 ) is x0 x  y0 y  r 2

Example. Find the equation of the tangent line to circle  : x2  y 2  6 x  4 y  12  0 at T (1,1) .

Solution. Since the equation of the tangent line to the circle  : x2  y 2  2mx  2ny  p  0 at

the point T ( x0 , y0 ) is given by x0 x  y0 y  m  x0  x   n  y0  y   p  0 , then the equation of

the tangent line to the circle  : x2  y 2  6 x  4 y  12  0 at the point T (1,1) is 4 x  3 y  7  0 .

1.8. 5. 2. THE ELLIPSE

5.2.1. Canonical equation

Def . An ellipse is the locus of points in a plane whose distances from two fixed points in the
plane has a constant sum. The two fixed points are the foci of the ellipse, and the distance
between them is called the focal length.

Let denote the left focus and right focus of the ellipse by F1 and F2 respectively.

Let M ( x, y) be an arbitrary point of the ellipse and choose the system of coordinates in such a

way F1 ( c, 0) and F2 (c, 0) .The distance between the two foci is d ( F1 , F2 )  F1 F2  2c .

The distances from the point M to the foci F1 and F2 are called focal radii and are denoted by

r1  d ( F1 , M ) and r2  d ( F2 , M ) respectively. According to the definition of the ellipse, the sum


of focal radii is constant. Denoting this constant by 2a , we have:

By Theoneste Hakizimana
36

d ( F1 , P)  d ( F2 , P)  2a (5.14)

From the triangle F1F2 M , using the triangle inequality we deduce: a  c.

Using distance formula between two points, the equation (5.14) is written

( x  c ) 2  y 2  ( x  c ) 2  y 2  2a

Writing the above equation as: ( x  c) 2  y 2  2a  ( x  c) 2  y 2

and squaring both sides, we obtain

x 2  2cx  c 2  y 2  4a 2  4a ( x  c) 2  y 2  x 2  2cx  c 2  y 2

which simplifies to: a ( x  c) 2  y 2  a 2  cx

Squaring both sides gives: a2 ( x2  2cx  c2  y 2 )  a 4  x2  2a 2 cx  c 2 x 2

Which can be written in the form: (a2  c2 ) x2  a2 y 2  a2 (a2  c2 )

Since a  c  0 , setting b2  a2  c2 .The above equation becomes b2 x2  a 2 y 2  a 2 b2

Dividing both sides by a2 b2 yields

x2 y 2
 1 (5.15)
a 2 b2

The equation (5.15) is called canonical or standard equation of the ellipse.

x2 y 2
The equation   1 can also be written explicitly in the form
a 2 b2

b
y a2  x2 (5.16)
a

The line through the foci is the focal axis.

By Theoneste Hakizimana
37

The point on the focal axis midway between the foci is the center.

A line segment with end points on an ellipse is a chord of the ellipse.

The chord lying on the focal axis is the major axis of the ellipse.

The chord through the center perpendicular to the focal axis is the minor axis of the ellipse.

The points where the ellipse intersects its axes are the vertices of the ellipse.

The length of the major axis is 2a, and the length of the minor axis is 2b.

The number a is the semi major axis length, and b is the semi minor axis length.

The ellipse is symmetric to both x -axis and y -axis. It is also symmetric w.r.t the origin.

c
The ratio e  is called eccentricity. The noun eccentricity comes from adjective eccentric
a
which means off- center. The shape of ellipse depends on the value of its eccentricity.

Note: Ellipses with highly off-center foci are elongated and have eccentricity close to 1 and
ellipses with foci near to the center are almost circular and have eccentricity close to 0.

When the eccentricity equals 0, the ellipse degenerates into a circle.

The chord through a focus and perpendicular to the major axis is called latus rectum which is

b2
Latin for” right chord”. Its length is 2 p  2
a

a a
The lines (d1 ) : x   and (d 2 ) : x  are directrices of the ellipse.
e e

By Theoneste Hakizimana
38

5.2.2. Analytic expressions of focal radii

Let find the analytic expressions of the focal radii r1  d ( F1 , M ) and r2  d ( F2 , M ) respectively.

Since F1 (c,0); M ( x, y) , by distance formula between two points, we have,

 x2   b2  c2
r12  ( x  c) 2  y 2  ( x  c) 2  b 2 1  2   1  2  x 2  2cx  c 2  b 2 = 2 x 2  2eax  a 2  (ex  a ) 2
 a   a  a

Since x  a, we have ex  a and therefore r1   a  ex 


2
 a  ex

Similarly, the other focal length is r2  a  ex . To summarize, we have

 r1  a  ex
 (5.17)
r2  a  ex

x2 y 2
The parametric equations of the ellipse E : 2  2  1 may be written
a b

x  a cos t ; y  b sin t (5.18)

x2 y 2
The equation of the tangent to the ellipse 2  2  1 at the point T ( x0 , y0 ) is
a b

By Theoneste Hakizimana
39

x0 x y0 y
 2 1 (5.19)
a2 b

Example. Given the ellipse governed by the equation E : 16x 2  25 y 2  400  0 . Find:

a. The foci b. The vertices c. The eccentricity d. The equations of directrices

16
e. The parametric equations f. The equation of the tangent line at the point T (3, )
5

Solution. The standard equation of the ellipse E : 16 x 2  25 y 2  400 is written

x2 y2
E:   1 .We have, a  5; b  4 and c  a 2  b 2  9  3
52 42

a. The foci of the ellipse are F1,2   c, 0    3, 0 

b. The vertices are A1,2   a, 0    5, 0  ; B1,2  0, b    0, 4 

c 3
c. The eccentricity is given by e    0.6
a 5

a2
d. The equations of directrices are given by  d1,2  : x   We have  d1,2  : x  
25
c 3

e. The parametric equations of the ellipse are: x  5cos t; y  4sin t

 16  xx y y
f. Since ( x0 , y0 )   3,  , using the formula 02  02  1 of the equation of the tangent
 5 a b
line to the ellipse, we have: 3x  5 y  25  0

1.9. 5.3. HYPERBOLA

Def.A hyperbola is the locus of points in a plane whose distances from the two fixed points in
the plane have a constant difference. The two fixed points are the foci and the distance
between them is called the focal length.

By Theoneste Hakizimana
40

Let denote the left focus and right focus of the hyperbola by F1 and F2 respectively.

Let M ( x, y) be an arbitrary point of the hyperbola and choose the coordinate system in such a

way F1 ( c, 0) and F2 (c, 0) .The distance between the two foci is d ( F1 , F2 )  F1 F2  2c .

The distances from the point M to the foci F1 and F2 are called focal radii and are denoted by

r1  d ( F1 , M ) and r2  d ( F2 , M ) respectively. According to the definition of the hyperbola, the

absolute value of their difference is constant.

Denoting it by 2a , we have

d ( F1 , M )  d ( F2 , M )  2a (5.20)

By the triangle inequality, we have: a  c .In terms of coordinates, (5.20) is written

x  c 2  y 2   x  c 2  y 2  2a

Writing the above equation as ( x  c ) 2  y 2  ( x  c ) 2  y 2  2 a

Transposing one radical ( x  c) 2  y 2  ( x  c) 2  y 2  2a

Squaring and collecting terms cx  a 2  a ( x  c) 2  y 2

Squaring and simplifying: (c2  a2 ) x2  a2 y 2  a2 (c2  a2 )

By Theoneste Hakizimana
41

Since c  a  0 , let b2  c2  a2

The above equation becomes: b2 x2  a2 y 2  a2b2

Dividing both sides by a2 b2 leads to

x2 y2
 1 (5.21)
a 2 b2

The equation (5.21) is called the canonical or standard equation of the hyperbola.

x2 y 2
The equation   1 can also be written explicitly as
a 2 b2

b 2 a
y x  a2 or x   y 2  b2 (5.22)
a b

The hyperbola has two axes of symmetry: one axis intersects the curve and is called the
transverse axis of symmetry. The other does not intersect the hyperbola and is called non-
transverse or conjugate axis of symmetry. The points A1 (  a, 0) and A2 (a, 0) are called vertices.

The numbers a and b are called semi transverse and semi conjugate axis length of the
hyperbola respectively. The point O(0, 0) is called the center of the hyperbola and it is the point
of intersection of the axes of symmetry. The hyperbola has two branches. The points for which
x  a form the right-hand branch and the points for which x  a form the left-hand branch.

c
The number e  is the eccentricity of the hyperbola (e  1) . The chord through a focus and
a
b2
perpendicular to the transverse axis is called latus rectum. Its length is 2 p  2
a

a a
The straight lines (d1 ) : x   and (d 2 ) : x  are called directrices.
e e

b b
The lines y  x and y  x are called the asymptotes of the hyperbola.
a a

By Theoneste Hakizimana
42

Note. The asymptotes of a hyperbola satisfy the following property: The product of distances
a 2b 2
from any point of hyperbola to its asymptotes is constant and equals 2 .
a  b2

5.2.2. Analytic expressions of focal radii

Let find the analytic expressions of the focal radii r1  d ( F1 , M ) and r2  d ( F2 , M ) respectively.

Since F1 (c,0); M ( x, y) , then the length r1 can be found by distance formula:

 x 2   b2  c2
r12  ( x  c)2  y 2  ( x  c)2  b 2  2  1   2  1 x 2  2cx  c 2  b 2 = 2 x 2  2eax  a 2  (ex  a ) 2
a  a  a

In the same way, we can show that: r22  (ex  a ) 2

Since | ex || x | a, we have

 a  ex, x  0
 r1  a  ex  
 a  ex, x  0
 (5.23)
r a  ex, x  0
 ex  a  
2 a  ex, x  0

From (5.23), we observe that r1  r2  2a

By Theoneste Hakizimana
43

x2 y 2  x y  x y 
Writing the equation of the hyperbola H : 2  2  1 into the form       1 and
a b  a b  a b 
x y x y 1 x y x y 1
setting t   , we have   . Solving the system,   t ;   , the parametric
a b a b t a b a b t
x2 y 2
equations of the hyperbola H : 2  2  1 may be written
a b

a  1 b  1
x t  , y  t   (5.24)
2 t 2 t 

x2 y 2
In trigonometric forms, the parametric equations of the hyperbola H :   1 are written
a 2 b2

x  acht; y  bsht or x  a sec t; y  b tan t (5.25)

x2 y 2
The equation of the tangent to the hyperbola H :   1 at the point T ( x0 , y0 ) is given by
a 2 b2

x0 x y0 y
 2 1 (5.26)
a2 b

Example: Given the hyperbola H : 9 x 2  16 y 2  144  0 . Find:

a. The foci b. the vertices c. the eccentricity d. The equations of directices.

e. The parametric equations f. The equations of asymptotes

9
g. The equation of the tangent line at the point T (5, )
4

x2 y2
Solution. The standard equation of the hyperbola is H :  1
4 2 32

In this case a  4, b  3 and c  a 2  b 2  5

a. The foci of the hyperbola are: F1,2  c, 0   F1,2  5, 0 

b. The vertices are A1,2  a, 0   A1    4, 0 

By Theoneste Hakizimana
44

c 5
c. The eccentricity is given by e  
a 4

a2
d. The directrices are governed by  d1,2  : x     
16
c 5

e. The parametric equations are obtained using the formula x  acht; y  bsht

We have, x  4cht; y  3sht

b 3
f. The equation of asymptotes are given by, y   x .Applying, we get y1,2   x
a 4

x0 x y0 y
g. The equation of the tangent line to the hyperbola is  2 1
a2 b

Applying the above formula yields 45x  y 144  0

1.10. 5.4. PARABOLA

Def . A parabola is the locus of points in a plane equidistant from a fixed line and a fixed point
not on the line. The fixed point F is called focus and the fixed line (d ) is the directrix of the
parabola. The distance from the focus to the directrix is the focal parameter.

Let M ( x, y) be an arbitrary point of the parabola and choose the system of coordinates in such

p  p
a way F  , 0  and its directrix is the vertical line  d  : x   0; p  0,
2  2

From definition of the parabola, we have   d (M , (d ))  d ( M , F )

By Theoneste Hakizimana
45

p p
Since d ( M , (d ))  x  and d ( M , F )  ( x  ) 2  y 2
2 2

p p
We have, x  = ( x  )2  y 2
2 2

Squaring both sides of the above equality and simplifying, we get

y 2  2 px (5.27)

The equation (5.27) is called the canonical equation of the parabola.

The canonical equation may explicitly be written : y   2 px

All the points of the parabola y 2  2 px lie on the right of y -axis. The x - axis is the axis of
symmetry and is called the focal axis. The point O(0, 0) is the vertex of the parabola. The chord
through the focus and perpendicular to the focal axis is called latus rectum. Its length is 2 p.

The number p is called the parameter of the parabola and it regulates its opening.

p
The distance d (O, F )  is the focal distance.
2

d (M , F )
The number e   1 is called eccentricity of the parabola.
d (M , d )

The parametric equations of the parabola y 2  2 px may be written

By Theoneste Hakizimana
46

t2
x ; y t (5.29)
2p

Or in trigonometric form: x  2 p cot 2 ; y  2 p cot  (5.30)

The equation of the tangent to the parabola y 2  2 px at the point T ( x0 , y0 ) is

yy0  p( x  x0 ) (5.31)

Example

Given the parabola P : 3 y 2  8x  0 . Find:

a. The focus b. The equation of the directrix c. The parametric equations

2 4
d. The equation of the tangent line to the parabola at the point T  , 
3 3

8 8 4
Solution.The standard equation of the parabola is y 2  x . Since 2 p  , we have, p 
3 3 3

p  2 
a. The focus is given by: F  , 0   F  , 0 
2  3 

p 2
b. The equation of the directrix is  d  : x    d  : x  
2 3

3t 2
c. The parametric equations are x  ;y t
8

d. Applying the formula of the equation of the tangent line to the parabola yy0  p( x  x0 ) ,

2 4
since ( x0 , y0 )   ,  , the equation of the tangent line is 3x  3 y  2  0
3 3

By Theoneste Hakizimana
47

1.11. 5.5. FOCUS-DIRECTRIX-ECCENTRICITY RELATONSHIP

x2 y 2
 If ri is the distance from an arbitrary point M ( x, y) of ellipse   1 to one of its
a 2 b2
foci Fi and d i is the distance from the same point to the corresponding directrix, then

ri r
the ratio is a constant and equals the eccentricity e of the ellipse i.e. i  e ; i  1, 2
di di

a
Proof. Consider the left focus F1 ( c, 0) and its corresponding directrix (d1 ) : x  
e

The respective distances from the point M ( x, y) of the ellipse to the directrix (d1 ) and the focus

a
F1 are : d1  x  ; r1  a  ex (5.32)
e

r1 a  ex
From (5.32), we can write  e
d1 x  a
e

a
Similarly, if we consider the right focus F2 (c, 0) and its corresponding directrix (d 2 ) : x  , we
e
r2 a  ex
obtain  e
d2 a  x
e

x2 y 2
 If ri is the distance from an arbitrary point M ( x, y) of hyperbola   1 to 1 of its
a 2 b2
foci Fi and d i is the distance from the same point to the corresponding directrix, then

ri r
the ratio is a constant and equals the eccentricity e of the hyperbola: i  e, i  1, 2 ;
di di

To summarize, the ellipse, hyperbola and the parabola can be defined in terms of focus,
directrix and eccentricity as the set all points in the plane for which the ratio of the distance r
from any arbitrary point of the conic to a certain fixed point (focus) and the distance d from the
same point of the locus to a certain fixed line (directrix) is a constant value (eccentricity

By Theoneste Hakizimana
48

d ( P, Fi ) r
 e (5.35)
d ( P,  d ) d

If 0  e  1, we have an ellipse; if e  1 , we have a hyperbola and if e  1 , we have a parabola.

1.12. 5.6. POLAR EQUATIONS OF CONICS

Let L denote one of three conics: ellipse, hyperbola and parabola.

For the hyperbola, only one of its branches will be considered.

Let F be the focus of the conic and (d ) its corresponding directrix. For the case of the
hyperbola, we shall consider the focus and the directrix which are closer to the chosen branch.

Let introduce the polar coordinates (  , ) in such a way the pole coincides with the focus F
and the polar axis be perpendicular to the directrix (d ) (fig.).

r
To obtain the polar equations of the conic L we have to use the relation  e , where r is the
d
length of the focal radius of a point on the conic and d the distance from that point to the
corresponding directrix. Assuming that the pole coincides with the focus we have

r (5.36)

From fig. , we have, d  QM  DN  DF  FN

Where, Q is the projection of M on the directrix (d ) . Since FN   cos  , we can write

d  DF   cos  (5.37)

By Theoneste Hakizimana
49

Let P be a point of the conic L such that the segment FP is perpendicular to the axis of the curve
L and p be the length of FP i.e. p is the half of the latus rectum of L (focal parameter).

Using (5.36) which is satisfied for any point of L , we have

FP FP p
 e  SP  
SP e e

Where, S is the projection of P on the directrix (d ) ).

p
Since SP  DF , we have DF  and the relation (5.37) can be written
e

p
d   cos  (5.38)
e

r   p
Since, d   , we have    cos  , which may be written as
e e e e

p
 (5.39)
1  ecos

Equation (5.39) is called the polar equation of conic.

Example. Write down the polar equations of the ellipse, hyperbola and parabola if their
respective equations are (i) E :16 x2  25 y 2  400 (ii) H : 9 x2 16 y 2 144  0

(iii) P : 3 y 2  8x  0

x2 y 2
Solution:(i)The canonical equation of the ellipse 16 x2  25 y 2  400 is   1.
52 4 2

b 2 16 c 3 p
We have, p   , e   . Applying the formula   , we obtain
a 5 a 5 1  ecos

16
5 16
 
1  cos  5  3cos 
3
5

By Theoneste Hakizimana
50

x2 y 2
(ii)The canonical equation of the hyperbola 9 x  16 y  144  0 is 2  2  1 .
2 2

4 3

b2 9 c 5
We have, p   ,e  
a 4 a 4

Applying the formula of the polar equation of a conic, we obtain,


9
9
 4 
1  cos  4  5cos 
5
4

8
(iii)The standard equation of the parabola is y 2  x
3

8 4
We have, 2 p   p  . The polar equation of the parabola is
3 3

4
p 3 4
  
1  cos  1  cos  3  3cos

By Theoneste Hakizimana
51

Unit6: GENERAL EQUATION OF SECOND ORDER CURVES

1.13. 6.1. SIMPLIFICATION OF GENERAL EQUATION

In affine Euclidean plane endowed with an orthonormal frame F (O; e1 , e2 ) , the general

equation of a second order curve is governed by

( x, y ) : a11 x 2  2a12 xy  a22 y 2  2a1 x  2a2 y  a0  0 (6.1)

Where, the coefficients aij are not all zero i.e. a11  a22  a12  0

P1. By means of a rotation and a translation, eq (6.1), can be reduced to one of the3 forms:

I: 1 x 2  2 y 2  a0  0,  12  0 

II: 2 y 2  2a1 x  0,  2 a1  0 

III: 2 y 2  a0  0,  2  0 

Proof. Passing from the system Oxy to the system Oxy under the rotation through the angle

 x  cos   sin    x 
 described by        , the general equation (6.1) becomes
 y   sin  cos    y

( x, y) : a11


 x2  2a12
 xy  a22
 y2  2a1x  2a2 y  a0  0

Where,

  a11 cos 2   2a12 cos  sin   a22 sin 2 


a11

  a11 sin 2   2a12 cos  sin   a22 cos 2 


a22

1
  (a22  a11 ) cos  sin   a12 (cos 2   sin 2  ) 
a12 (a22  a11 ) sin 2  a12 cos 2
2

a1  a1 cos   a2 sin ; a2  a1 sin   a2 cos ; a0  a0

By Theoneste Hakizimana
52

2a12
Assuming a12  0 , and taking angle  defined by: tg 2  ,
a11  a22

The coefficient a12 of the cross term is eliminated and the equation takes the form

  x, y  : 1 x2  2 y2  2a1 x  2a2 y  a0  0 (6.2)

Where, 1  a11  ; a0  a0


 ; 2  a22

Performing translation of axes of coordinates, let examine all possible forms that equation (6.2)
may take in terms of different values of i , s and coefficients ai, s .

I. 12  0

After dropping the accents, the equation (4.2) can be written


2 2
 a   a   a2 a2 
1  x  1   2  y  2    a0  1  2   1 x2  2 y2  a0  0
 1   2   1 2 

a1 a2 a12 a22
Where, x  x  ; y  y  ; a0  a0  
1 2 1 2

II. 2 a1  0, We have,

2
 a  a2
2  y  2   2a1 x  a0  2  2 y2  2a1 x  a0  2 y2  2a1 x  0
 2  2

ao a a2
Where, x  x  ; y  y  2 ; a1 =a1 ; a0  a0  2
2a1 2 2

III . 2  0; We have,

2
 a2  a22
2 y  2a2 y  a0  2  y    a0   2 y2  a0  0
2

 2  2

a2 a2 2
Where, y  y  ; a0  a0 
2 2

By Theoneste Hakizimana
53

1.14. 6.2. CLASSIFICATION OF SOC

The classification of SOC can be made in terms of the coefficients of the reduced forms as:

I. 1 x 2  2 y 2  a0  0

a0 a0
i) If a0  0, divide by  a0 and put  a2 ;  b 2
1 2

The signs being so chosen that a, b are real. This gives the following 3 types

x2 y 2
I1. 2  2  1: real ellipse
a b
x2 y 2
I 2 . 2  2  1: imaginary ellipse
a b
x2 y 2
I 3 . 2  2  1: hyperbola
a b

ii) If a0  0, the reduced forms are

x2 y 2
I4.   0 : intersecting imaginary lines
a 2 b2
x2 y 2
I 5 . 2  2  0 : intersecting real lines
a b

II : 2 y 2  2a1 x  0. The equation of the curve takes the form

y 2  2 px : parabola

III . 2 y 2  a0  0

If a0  0, according to the sign of a0 , the equation may be written

III1. y 2  a 2 : two parallel lines


III 2 . y 2  a 2 : two imaginary parallel lines

If a0  0, we have

III 3 . y 2  0 : one line counted twice

By Theoneste Hakizimana
54

1.15. 6.3. CENTER OF A QUADRIC CURVE

Def . A point C ( x0 , y0 ) is called the center of the curve (6.1), if the origin of coordinates is

placed at that point; the equation (6.1) loses its linear part i.e. the coefficients a1 , a2 vanish.

The translation equations of the origin of coordinates to the point C ( x0 , y0 ) are governed by

x  x  x0 ; y  y  y0

Substituting into the equation (6.1), we have

( x, y) : a11


 x2  2a12
 xy  a22
 y2  2a1x  2a2 y  a0  0 (6.3)

  a11; a12
Where, a11   a12 ; a22  a22 ; a0  ( x0 , y0 )

a1  a11 x0  a12 y0  a1; a2  a12 x0  a22 y0  a2

From the definition of the center of a curve, C ( x0 , y0 ) is the center of  iff a1  a2  0 .

This condition is equivalent to the system of equations

1
  (x , y )  0
a11 x0  a12 y0  a1  0 2 x 0 0
   (6.4)
a12 x0  a22 y0  a2  0 1  (x , y )  0

2
y 0 0

( x, y ) ( x, y )
Where,  x  ; y 
x y

The relations (6.4) imply that centers of the quadric curve (6.1) are the intersecting points of
the lines (d1 ) : a11 x  a12 y  a1  0 and (d2 ) : a21 x  a22 y  a2  0 .Three cases may occur

(i) The two lines are concurrent, the curve has unique center and the conic is called a
central conic
(ii) The two lines are parallel, and the curve has no center

By Theoneste Hakizimana
55

(iii) The two lines coincide, and the curve has a line of centers.
Pr 2. A quadric curve ( x, y )  a11 x 2  2a12 xy  a22 y 2  2a1 x  2a2 y  a0  0

(i) has unique center if and only if   0


(ii) has no center if and only if   0,   0
(iii) has a line of centers if and only if     0
a11 a12 a1
a11 a12
 ;   a12 a22 a2
a12 a22
a1 a2 a0

Proof.

(i) ( x, y) has unique center if and only if (6.4) possesses unique solution and this occurs if

a11 a12
and only if  , which is equivalently to a11a12  a122    0
a21 a22

(ii) ( x, y) has no center if and only if (6.4) is inconsistent system i.e.

a11 a12 a1
  , which is equivalent to   0 ;   0
a21 a22 a2

(iii) ( x, y) has a line of centers if and only if (6.4) possesses many solutions i.e.

a11 a12 a1
  , which is equivalent to     0
a21 a22 a2

Note: Central conics are two ellipses, the hyperbola and two concurrent lines. The parabola is a
non-central conic. The conics with a line of centers are 2parallel lines and a line counted twice.

Example

By means of rotation and translation, reduce to standard form the following SOC.

a) ( x, y) : 5x2  4 xy  8 y 2  32 x  56 y  80  0

b)   x, y  : 7 x 2  16 xy  23 y 2  14 x  16 y  218  0

c)   x, y  : 4 x 2  4 xy  y 2  2 x  14 y  7  0

By Theoneste Hakizimana
56

Solution
a) The equation   x, y  : 5 x 2  4 xy  8 y 2  32 x  56 y  80  0 can be written

 5 2 16  x 
  x, y  :  x y 1  2 8 28  y   0
 16 28 80  1 

5 2
  36  0 . The conic is elliptic type
2 8
The centre of this conic is the solution of the system of equations:

5 x  2 y  16

2 x  8 y  28

Solving, we find that the centre of the conic is C (2,3)

Effect of translation of axes

To eliminate first degree terms of the equations, we must use the translation equations

x  x  2; y  x  3

Substituting into the equation of the conic yields


  x, y  : 5 x2  4 xy  8 y2  36  0

Effect of rotation of axes


To find the angle of rotation, let us use the formula
2a12 4 4 2 tan 
tan 2     . Using the formula, tan 2  ,
a11  a22 5  8 3 1  tan 2 

we have, 2 tan 2   3 tan  2  0


35  1
Solving this quadratic equation, we obtain: tan    2,  
4  2
Using tan   2 and applying the rotation formula

 1 2 

 x   cos  sin    x    x   5 5   x  

 y  sin  cos   y  .We have,  y    1   y 
         2
 5 
 5

By Theoneste Hakizimana
57

Substituting into the equation of ( x, y)  0 , the cross term is removed, and we get

  x, y  : 9 x  4 y2  36

Dividing both sides of equality by 36, we obtain the canonical equation of the ellipse
x2 y2
  x, y  :  2 1
22 3
b) In matrix form, the equation of the conic
  x, y  : 7 x 2  16 xy  23 y 2  14 x  16 y  218  0 is written

7 8 7   x 
  x, y  :  x y 1  8 23 8   y   0

 7 8 218 1 

7 8
Since    225  0 , we have a hyperbolic type.
8  23

7 x  8 y  7
The centre of the conic is obtained by solving: 
8 x  23 y  8
Solving, we find that the centre of the hyperbola is C 1, 0 

Effect of translation
Using translation equations, x  x  1; y  y and substituting into the equation   x, y  ,
the first-degree terms of conic are removed. We have
  x, y  : 7 x2  16 xy  23 y2  225  0

Effect of rotation
2a12 8
To find  , use tan 2   . This leads to the equation
a11 a 22 15
1
4 tan 2   15 tan  4  0 having two roots: tan   ; tan   4
4
 4 1 
 
 x  17 17   x 
Using the first root, the equations for rotations are:      
 y  1 4   y
 
 17 17 
Substituting into the equation of   x, y   , we get:   x, y  : 9 x2  25 y2  225  0

Dividing both sides of the equality by225, the standard equation of the hyperbola is

By Theoneste Hakizimana
58

x2 y2
  x, y  :  2 1
52 3
c) In matrix form, the equation of the conic   x, y  : 4 x 2  4 xy  y 2  2 x  14 y  7  0 is

 4 2 1  x 
 ( x, y ) :  x y 1  2 1 7   y   0
 1 7 7  1 

4 2
Since    0 , the conic is a parabolic type.
2 1
2a12 4
The angle of rotation  is given by: tan 2  
a11  a22 3
1
This leads to equation 2  tan    3 tan   2  0 .Solving, we get, tan  2 or tan   
2

2
1 2
Using tan   2 , we have cos   ;sin  
5 5
 1 2 
x   5 5   x 
The equations of the rotation are:     
 y  2 1   y
 5 
 5

Substituting into   x, y  , we get:   x, y  : 5 y2  6 5 x  2 5 y  7  0

2
 5 6 5 5
This equation can be written:   x, y  :  y     x  0
 5  5  5 

5 5
Performing the translation: x  x  ; y  y 
5 5
6 5
The standard form of the parabola is   x, y  : y  2  x 
5
Note. For practical purpose, in reducing the general equation of a conic to canonical form, it is
better for the case of a central conic to start with a translation followed by a rotation and for
the case of a non-central conic to start by a rotation then proceed with a translation.

By Theoneste Hakizimana
59

4. ORTHOGONAL INVARIANTS
In terms of quadratic and linear forms, the equation (6.1) may be written

( x, y) : q( x, y)  2l ( x, y)  a0  0 (6.5)

Where, q ( x, y )  a11 x 2  2a12 xy  a22 y 2 and l ( x, y)  a1 x  a2 y

are respectively the quadratic and linear forms associated with the SOC.

In matrix notations, (6.1) can be written

( x, y) : X T AX  2BX  C  0 (6.6)

a a12  x
Where, A   11 is the matrix of q( x, y) , B   a1 a2  , X    and C  a0
 a12 a22   y

ˆ ˆ 0
( x, y ) : Xˆ T AX (6.7)

x  a11 a12 a1 


ˆ   ˆ  a2 
Where, X   y  and A   a12 a22
 1   a1 a2 a0 

Introducing a new coordinates system Oxy associated with the orthonormal frame

F (O, e1 , e2 ) , the systems x, y and x, y are related by

 x   p11 p12 p1   x 
 x   p11 p12   x   p1    
 y   p         y    p21 p22 p2   y (6.8)
   21 p22   y  p2 
1   0 0 1  1 

p p12 
Where, P   11 is the transition matrix from the basis e1 , e2  to the basis e1 , e2
 p21 p22 

and OO  p1 e1  p2 e2 . Since the two bases are orthonormal, then the matrix P is orthogonal.

In the new coordinates system, the equation (6.7) is written

By Theoneste Hakizimana
60

 a1 1 a1 2 a1 


 a2 
( x, y) : Xˆ  Aˆ Xˆ   0 , A   a1 2
T
a22 (6.9)
 a1 a 2 a0 

The matrices A and A are related to the matrices A and A respectively by

 p11 p11 p1 
ˆ ˆ , Where Pˆ   p21
A  PT AP and Aˆ   Pˆ T AP  p22  
p2  . Let   det Aˆ and   det  A 
 0 0 1 

Pr .Under the transformation X   PX ,  and  are not modified i.e.;    ;     .

Proof.   det Aˆ   det( Pˆ T AP


ˆ ˆ )  det Aˆ  (det Pˆ ) 2  det Aˆ  (det P ) 2  det Aˆ  

   det A  det( PT AP)  det A  (det P)2  det A   .Thus,  and  are orthogonal invariants.

Pr. The trace of the matrix A , denoted by S  a11  a22 is also an orthogonal invariant i.e. S  S 

 a    p11
a12 p21   a11 a12   p11 p12 
Proof: A   11 

 a12    p12
a22 p22   a12 a22   p21 p22 

  p11 ( p11a11  p21a12 )  p21 ( p11a12  p21a22 ) ; a22


a11   p12 ( p12 a11  p22 a12 )  p22 ( p12 a12  p22a22 )

S   a11
  a22
  a11 ( p112  p122 )  a22 ( p21
2
 p22
2
)  2a12 ( p11 p21  p12 p22 )  a11  a22  S .

Pr. The characteristic polynomial of the quadratic part of  is an orthogonal invariant.

Proof: Let denote the characteristic polynomial of A by PA ( ).

1 0 
By definition, PA ( )  det( A   I 2 )   2  S    ; I 2   ,
0 1 

PA ( )  det( A   I 2 )  det( PT AP  PT  P)  det( PT ( A   I 2 ) P)  det( A   I 2 )  PA ( )

By Theoneste Hakizimana
61

Note. The invariance of PA ( ) results also from the invariance of the quantities S and  . The

roots 1 and 2 of the characteristic polynomial PA ( ) are also orthogonal invariants.

a22 a2 a11 a1
Pr.Under rotation, the quantity K    m11  m22 is not modified.
a2 a0 a1 a0

Proof: The rotation of the axes of the co-ordinates about the origin through the angle 

 x  cos   sin    x 
governed by     , transforms the coefficients of SOC according to
 y   sin  cos    y

a1  a1 cos   a2 sin ; a2  a1 sin   a2 cos ; a0  a0

Using the above relations and the invariance of S , we get:

K   a0 (a11
  a22
 )  a12  a22 
a0 S   (a1 cos   a2 sin  ) 2  (a1 sin   a2 cos  ) 2  a0 (a11  a22 )  a12  a22  K

Note. K is not also modified under reflection in one axis with respect to another.In general,
under translation, K is not invariant and for this reason, it is called a semi orthogonal invariant.

Pr. If     0 , then K is an orthogonal invariant.

6.5. QUADRIC CURVES IN TERMS OF THEIR ORTHOGONAL INVARIANTS


By means of rotation and translation the general equation of a SOC is reduced to 1 of 3 forms:

1 x 2  2 y 2  a0  0 (6.10)

2 y 2  2a1 x  0 (6.11)

2 y 2  a0  0 (6.12)

The matrix  associated with each of the three forms is written respectively

1 0 0 0 0 a1  0 0 0
I . Aˆ   0 2 0  II . Aˆ   0 2 0  III . Aˆ  0 2 0 
 0 0 a0   a1 0 0  0 0 a0 

By Theoneste Hakizimana
62

 The form I is characterized by:   12  0 ;   12 a0 .The equation (6.10) is written


1 x 2  2 y 2  0 (6.13)

 The form II is characterized by:   0 ;   2 a12  0 ; S  2


From the expressions of  and S , yields a1  
S


y 2  2 x0 (6.14)
S3

 The form III is characterized by     0 .

We know that if     0 , then K is an orthogonal invariant. Since K  2 a0 ; S  2 , then

K K
K  a0 S and the equation (6.12) is written : Sy 2   0 or y 2  2  0
S S
(6.15)

By Theoneste Hakizimana
63

Various cases Conditions on the Canonical equation curve no


invariants:  ;  ; S ; K

I  0 0 S  0 1 x 2  2 y 2 

0 x2 y 2 (1)
  1
a 2 b2
a11 x  a22 y  a0  0
2 2

S  0  x2 y 2 (2)
1 x 2  2 y 2  0   1
 a 2 b2

0  x2 y2 (3)
1 x 2  2 y 2  0  0
 a 2 b2

0  x2 y 2 (4)
1 x 2  2 y 2  0   1
 a 2 b2

0  x2 y 2 (5)
1 x 2  2 y 2  0  1
 0  a 2 b2

II  0 0  y 2  2 px (6)
y2  2 x
S3
a22 y 2  2a1 x  0

III  0 0 K 0 K y2  a2 (7)


y2  0
S2
a22 y 2  a0  0
K 0 K y 2  a 2 (8)
y2  0
S2

K 0 K y2  0 (9)
y2  0
S2

In terms of the invariant  , there are three types of second order curves: ellipse:   0 ;
hyperbola:   0 and parabola   0 . If   0 , degenerate curves occur.

By Theoneste Hakizimana
64

Example.

Using the technique of orthogonal invariants, reduce to the standard form the equations of
following quadric curves

a) ( x, y) : 5x2  4 xy  8 y 2  32 x  56 y  80  0
b)   x, y  : 7 x 2  16 xy  23 y 2  14 x  16 y  218  0
c)   x, y  : 4 x 2  4 xy  y 2  2 x  14 y  7  0
d)   x, y  : x 2  5 xy  4 y 2  x  2 y  2  0

Solution

a) Let find the orthogonal invariants of the curve


 5 2 16  x 
( x, y ) :  x 
y 1  2 8 28  y   0
 16 28 80  1 

5 2
Since    36 , we have a conic of elliptic type
2 8

5 2 16
Computing,   2 8 28  1296 .We obtain a non-degenerate ellipse
16 28 80

5 2
PA       2  13  36
2 8
The Eigenvalues of A are the roots of the characteristic polynomial
 2  13  36     9    4   0 .Solving, we get, 1  9 ; 2  4
The standard form of the conic in terms of orthogonal invariants is given by

  x, y  : 1 x2  2 y2  0.

Applying the formula, we obtain   x, y  : 9 x2  4 y2  36  0 ;

x2 y2
The canonical equation of this ellipse is   x, y  :  2 1
22 3

By Theoneste Hakizimana
65

b) The curve can be written as:


 7 8  7  x 
  x, y  :  x 
y 1  8  23  8   y   0
 7  8  218 1 

7 8
Since    225 ,We have conic of hyperbolic type
8 23

7 8 7
Computing,   8 23 8  50625 .We obtain a non-degenerate hyperbola
7 8 218

7 8
PA       9  25
8  23  

The roots are 2  9; 2  25 . Using the formula   x, y  :  x2  2 y2   0,

x2 y2
  x, y  : 9 x2  25 y2  225  0    x, y  :  2 1
52 3

c) The equation of the curve is given by:


 4 2 1  x 
4 2
  x, y  :  x y 1  2 1 7   y   0 . Since    0 : parabolic type
2 1
 1 7 7  1 

4 2 1
Computing,   2 1 7  225 .We obtain a non- degenerate parabola
1 7 7

 225
S  a11  a22  4  1  5 . Using the formula, y2  2 3
x ,we obtain, y  2  2 x  .
s 53
5
  x, y  : y2  6 x
5
 1 2.5 0.5  x 
d )   x, y  :  x 
y 1  2.5 4 1   y   0
 0.5 1 2  1 

By Theoneste Hakizimana
66

1 2.5
Since    2.25 0 ,We have a hyperbolic conic
2.5 4

1 2.5 0.5
Computing,   2.5 4 1  0 .We obtain a degenerate hyperbola
0.5 1 2

1   2.5  5  34  5  34  5  34 5  34
PA             ; 1  ; 2  
2.5 4    2  2  2 2


Using the formula,   x, y  : 1 x2  2 y2   0,

5  34 2 5  34 2
We have,   x, y  : x  y  0
2 2
The curve is a degenerate hyperbola into two intersecting real lines

Note. Since   0 , we have a degenerate conic, and we can find its components

by solving the quadratic equation related to one of the two variables as follows:

x2  5( y 1) x  4 y 2  2 y  2  0,

Solving with respect to the variable x , we obtain

5 y  1  (3 y  3)
x1,2  or x1  4 y  2; x2  y  1
2

The equation of the degenerate curve is:  x  4 y  2  ( x  y  1)  0

The curve is degenerated into two concurrent lines

 d1  : x  4 y  2  0 and  d 2  : x  y  1  0

Example

In terms of the real parameter k determine the type of the following family of quadric curves
governed the equation  k ( x, y ) : x 2  2kxy  y 2  1

Solution:In matrix notation, the equation of the curve may be written

By Theoneste Hakizimana
67

1 k 0   x 

 k  x, y  :  x y 1  k 1 0   y   0 ;   1  k 2 ;   k 2  1;  1

 0 0 1 1 

The roots of characteristic polynomial


1  k
PA         (1  k )    (1  k )  are 1  1  k; 1  1  k
k 1 
If     1  k 2 0 or equivalently 1 k 1 , then we have a family of ellipse whose
canonical equation can be reduced to the form:   x, y  : (1  k ) x2  (1  k ) y2  1

If k  0 , we have a circle of equation  : x 2  y 2  1 .

If     1  k 2 0 or equivalently k    1,1 , then we have

a family of non-degenerate hyperbolas whose canonical equation can be reduced to the form

  x, y  : (1  k ) x2  (1  k ) y2  1

If     0 or equivalently k  1,1 , we have a family of degenerate parabolas.

If k  1 , we have a pair of parallel lines  d1  : x  y  1  0 and  d 2  : x  y  1  0

If k  1 , we have a pair of parallel lines  d1  : x  y  1  0 and  d 2  : x  y  1  0

By Theoneste Hakizimana
68

Unit7: ELEMENTARY PROBLEMS OF SOLID ANALYTIC GEOMETRY

Let    A3 , V , ,  be a 3D- affine Euclidean space equipped with an orthonormal frame


F (O; e1 , e2 , e3 ) and P be an arbitrary point of A3 whose vector position is : OP  xe1  ye2  ze3

7.1. Distance between two points

The distance between the points P1 ( x1 , y1 , z1 ) and P2 ( x2 , y2 , z2 ) is given by

d ( P1 , P2 )  PP
1 2  ( x2  x1 ) 2  ( y2  y1 ) 2  ( z2  z1 ) 2 (7.1)

(i) d ( P1 , P2 )  0;(d ( P1 , P2 )  0  P1  P2 ) (Non-negativity)

1.16. (ii) d ( P1, P2 )  d ( P2 , P1 ) (Symmetry)

1.17. (iii) d ( P1 , P3 )  d ( P1 , P2 )  d ( P2 , P3 ) (Triangle inequality)

7.2. Dividing of a line segment in a given ratio

Pr.Let Q( x2 , y2 , z2 ) and P ( x1 , y1 , z1 ) . If ( x, y, z ) are the co-ordinates of the point M dividing the

segment PQ in the ratio  , then

x1   x2 y   y2 z   z2
x ,y 1 ,z  1 (7.2)
1  1  1 

Note. Depending on the sign and value of  , the following cases may occur

If   0 , then M is inside the segment PQ ; if   0 , then M is outside of PQ


If   0 , then M  P ; if    , then M  Q
If   1 , the point M moves away indefinitely on the line carrying PQ

If M is the midpoint of PQ , then   1 and we obtain the mid-point formula


x1  x2 y  y2 z z
x ;y 1 ;z 1 2
2 2 2

The co-ordinates of the centroid G ( x, y, z ) of a system of material points Pi ( xi , yi , zi ) with

respective masses mi (i  1, 2,..., n) are given by:

By Theoneste Hakizimana
69

m1 x1  ...  mn xn m y  ...  mn yn m z  ...  mn zn


x ;y 1 1 ;z 1 1 (7.3)
m1  ...  mn m1  ...  mn m1  ...  mn

In particular, the co-ordinates of the centroid G ( x, y, z ) of a triangle

x1  x2  x3 y  y2  y3 z z z
A( x1 , y1 , z1 ), B( x2 , y2 , z2 ), C( x3 , y3 , z3 ) are x  ;y 1 ; z 1 2 3
3 3 3

and the coordinates of the centroid  ( x, y, z ) of the tetrahedron with vertices P1 ( x1 , y1 , z1 ) ,

P2 ( x2 , y2 , z2 ), P3 ( x3 , y3 , z3 ) and P4 ( x4 , y4 , z4 ) are:

x1  x2  x3  x4 y  y2  y3  y4 z z z z
x ;y 1 ; z 1 2 3 4
4 4 4

7.3. Volume of a tetrahedron

The volume of the tetrahedron whose vertices are P1 ( x1 , y1 , z1 ) , P2 ( x2 , y2 , z2 ), P3 ( x3 , y3 , z3 ) and

x1 y1 z1 1
x2  x1 y2  y1 z2  z1
1 x2 y2 z2 1 1
P4 ( x4 , y4 , z4 ) is given by: V     x3  x1 y3  y1 z3  z1
6 x3 y3 z3 1 6
x4  x1 y4  y1 z4  z1
x4 y4 z4 1

x1 y1 z1 1
x2 y2 z2 1
The points P1 , P2 , P3 , P4 are coplanar if and only if 0
x3 y3 z3 1
x4 y4 z4 1

Note. The area of the face of the tetrahedron whose vertices are P1 ( x1 , y1 , z1 ) , P2 ( x2 , y2 , z2 ) and

P3 ( x3 , y3 , z3 ) is given by

By Theoneste Hakizimana
70

2 2 2
e1 e2 e3 y1 z1 1 z1 x1 1 x1 y1 1
1  1 1
S   P1 P2 , P1 P3   x2  x1 y2  y1 z2  z1 = y2 z2 1  z 2 x2 1  x 2 y2 1
2 2 2
x3  x1 y3  y1 z3  z1 y3 z3 1 z3 x3 1 x3 y3 1

Example. The vertices of the tetrahedron are A2,1,0 , B1,3,5, C 6,3,4 and D  0, 7,8  .

Find:

i. The coordinates of the centroid of the face ABC and the tetrahedron ABCD

ii. The perimeter of the face ABC

iii. The volume of the tetrahedron

iv. The area of the face ABC

Solution.i. The centroid G of the face ABC of the tetrahedron is given by

 x  x  x y  yB  yC z A  zB  zC
 

G x, y , z   A B C , A
3 3
,
3
  7 
   3, ,3 
  3 

The centroid  of the tetrahedron ABCD is given by

 x  x  x  xD y A  yB  yC  yD z A  z B  zC  z D
 

 x, y , z   A B C
4
,
4
,
4
  9 17 
   , 0, 
 4 4

 1  4 5
ii. Since, AB   2  ; AC   2 ; BC   0  ; AB  30; AC  6 ; BC  26
   
 5   4  1

The perimeter P of the face ABC of the tetrahedron is given by

P  AB  AC  BC  30  6  26

By Theoneste Hakizimana
71

2 1 0 1
11 3 5 1 67
iii. The volume V of the tetrahedron is given by: V  
66 3 4 1 2
0 7 8 1

2 2 2
1 0 1 0 2 1 2 1 1
1
iv. The area of the face ABC is given by: S  3 5 1  5 1 1  1 3 1  170
2
3 4 1 4 6 1 6 3 1

7.2. THE PLANE

1. Various equations of a plane

In 3D, a plane is completely determined if one of the three following conditions is satisfied:

(1) Given its point and two linearly independent direction vectors
(2) Given its point and normal vector
(3) Given its three non-collinear points.

Pr. Let in the plane  , be given points P0 ( x0 , y0 , z0 ) , P( x, y, z) and two non-collinear vectors

 u1   u1   v1 
 
u  u2  u  u2 ; v  v2  , the equation of the plane  are given by:
 
   
 u3  u3   v3 

 : r  r0   u   v (7.4): Vector equation

 x   x0   u1   v1 
 :  y    y0    u2    v2 
      (7.5): Parametric equations
 z   z0  u3   v3 

x  x0 y  y0 z  z0
 : r  r0 , u , v  u1 u2 u3  0 (7.6): Analytic equation
v1 v2 v3

Expanding the determinant, we get:  : A( x  x0 )  B( y  y0 )  C ( z  z0 )  0

By Theoneste Hakizimana
72

u2 u3 u u1 u u
Setting, A  ;B  3 ; C  1 2 ; D   Ax0  By0  Cz0 , we have
v2 v3 v3 v2 v1 v2

 : Ax  By  Cz  D  0 (7.7) : General equation

If D  0 , the plane  passes through the origin of co-ordinates.


If A  0 (or B  0 , or C  0 ), the plane  is parallel to the axes Ox(Oy; Oz) respectively.

If A  B  0 (or A  C  0 , or B  C  0 ), the plane  is parallel to the planes Oxy; Oxz or


Oyz respectively.

Note .The vector N  u , v  is perpendicular to the plane  and is called normal vector to the

plane  .In terms of coordinates it may be written N   A B C 


T

In vector form the equation (5.8) can be written

 : r  r0 , N  0 (7.8)

The direction cosines of the vector N are

A B C
cos  ; cos  ; cos 
A  B C
2 2 2
A  B C
2 2 2
A  B2  C 2
2

Where,   ( N , Ox);   ( N , Oy );   ( N , Oz )

They satisfy the identity  cos     cos     cos    1


2 2 2

The normal equation of the plane  is given by


 : x cos   y cos   z cos   p  0 (7.9)

Where, p is the distance from the origin of the axes of the co-ordinates to the plan  .

Note: The equation of the plane  passing through three non collinear points

P1 ( x1 , y1 , z1 ), P2 ( x2 , y2 , z2 ) and P3 ( x3 , y3 , z3 ) is given by:

By Theoneste Hakizimana
73

x y z 1
x  x1 y  y1 z  z1
x y1 z1 1
: 1  x2  x1 y2  y1 z2  z1  0 (7.10)
x2 y2 z2 1
x3  x1 y3  y1 z3  z1
x3 y3 z3 1

In vector form (7.10) can be written:  : (r  r1 , r2  r1 , r3  r1 )  0 (7.11)

Note: If the plane  intersects the axes of coordinates Ox, Oy, Oz at the point
P1 (a,0,0), P2 (0, b,0) and P3 (0,0, c) respectively, then the equation of plane  passing
through these three points is governed by equation
x y z
:   1 (7.12)
a b c

The equation (5.12) is called the equation of the plane in intercept form.

2. Distance from a point to a plane

Pr.The distance from a point P0 ( x0 , y0 , z0 ) to the plane  : Ax  By  Cz  D  0 is given by

Ax0  By0  Cz0  D


d ( P0 ,  )  (7.13)
A2  B 2  C 2

3. Relative positions of two planes

Let  be the angle between the planes 1 : A1 x  B1 y  C1 z  D1  0 and

 2 : A2 x  B2 y  C2 z  D2  0 and  the angle between their respective normal vectors


T T
N1   A1 B1 C1  ; N 2   A2 B2 C 2 

If 0    900 , then    and If 900    1800 , then   1800   , where

N1 , N 2 A1 A2  B1B2  C1C2
cos    (7.14)
N1  N 2 A  B C
2 2 2
A2  B2  C2
2 2 2
1 1 1

Two planes  1 and  2 are concurrent if and only if  N1 , N 2   0 ,

By Theoneste Hakizimana
74

A1 B1 C1 D1
Two planes  1 and  2 coincide if and only if   
A2 B2 C2 D2

A1 B1 C1 D1
Two planes  1 and  2 are parallel if and only if   
A2 B2 C2 D2

Let 1  A1 x  B1 y  C1 z  D1  0 and  2  A2 x  B2 y  C2 z  D2  0 be two intersecting planes.

Def. A pencil of planes generated by the plans  1 and  2 is a family of the planes passing

through the intersection points of the plans  1 and  2 .It is given by the equation

A1 x  B2 y  C1 z  D1   ( A2 x  B2 y  C2 z  D2 )  0 (7.15)

All planes of the pencil can be obtained by varying the parameter  in the interval ,  .

If in (7.15) the parameter   0 , then we have the plane  1 .

Dividing both sides of (7.15) by  and evaluating the limit of the expression obtained when
  , we obtain the plane  2 . If   1 and  1 ;  2 are given by their normal equations, the

equation of the pencil gives the equations of the bisecting planes of the plans  1 and  2

Pr4.The distance between two parallel planes 1 : Ax  By  Cz  D1  0 and

D1  D2
 2 : Ax  By  Cz  D2  0 is given by: d (1 ,  2 )  (7.16)
A2  B 2  C 2

Note. The relative position of three planes 1 : A1 x  B2 y  C2 z  D1  0 ,

 2 : A2 x  B2 y  C2 z  D2  0 and  3 : A3 x  B2 y  C3 z  D3  0 can be treated algebraically by


solving the system of three equations in three variables

 A1 x  B1 y  C1 z  D1  0

 A2 x  B2 y  C2 z  D2  0
A x  B y  C z  D  0
 3 3 3 3

By Theoneste Hakizimana
75

 If the system is consistent and has unique solution, then the three planes are concurrent
in unique point.
 If the system is consistent and has many solutions, then the three planes coincide or
intersect in a straight line.
 If the system is inconsistent and has no solution, then either two distinct planes are
parallel or two planes taken in pairs are intersecting in two distinct parallel lines or the
three planes are parallel.

Example

Given the tetrahedron whose vertices are A  2,1, 0  ; B(1,3,5), C (6,3, 4) and D(0, 7,8)

1. Write down the equations of the faces ABC and ABD


2. Calculate the angle between the faces ABC and ABD

3. Calculate the length of height dropped from the vertex D to the face ABC

Solutiona.

i.a) The direction vectors of the face ABC are given by AB   1 2 5 ; AC   4 2 4


T

The vector equation of the face ABC is v  A  AB  AC

 x   2  1  4
The parametric equations of ABC are ABC :  y   1     2     2
     
 z  0  5   4

x  2 y 1 z
The Cartesian equation of the face ABC is given by: ABC :  1 2 5 0
4 2 4

Expanding the determinant, we get ABC   x  12 y  5z  10  0

b) The direction vectors of the face ABD are AB   1 2 5 ; AD   2 8 8


T T

The vector equation of ABD ABD is v  A   AB   AD

By Theoneste Hakizimana
76

 x   2  1  2
The parametric equations ABD are  y   1     2   B  8
     
 z  0  5   8 

x  2 y 1
The Cartesian equation of ABD is given by 1 2 5  0
2  8 8

ABD : 28x  y  6 z  55  0

ii. The angle  between the faces ABC and ABD is f the angle  between their corresponding

normal vectors. Since the normal vectors of ABC and ABD are N1   1 12 5
T
and

 N1 , N 2  70
N 2   28 1 6 respectively, we have, cos      100.800
T

N1 . N 2 139570

Since the angle  is obtuse e, we have,   1800 100.800  79.200

iii. The length hD of the height dropped from the vertex D equals the distance from the vertex

1 0  12   7   5  8  10 134
D(0, 7,8) to the face ABC . hD  d  D, ABC   
 1  122   5
2 2
170

By Theoneste Hakizimana
77

7.3. THE STRAIGHT LINE

1. Various equations of a straight line

A straight line is completely determined in the following two cases

(1) Given its point and direction vector (2)Given its two distinct arbitrary points

In a 3- dimensional space, a straight line (d ) can also be defined as intersection of two planes.

 A x  B1 y  C1 z  D1  0
(d ) :  1
 A2 x  B2 y  C2 z  D2  0 (7.17)

The equation of the straight line (d ) with direction vector u  u1 u2 u3  and passing
T


through the point P ( x0 , y0 , z0 ) with position vector r0 is written:

(d ) : r  r0  u (7.18) vector equation

 x   x0   u1 
(d ) :  y    y0    u2 
    (7.19 parametric equations
 z   z0  u3 

Excluding the parameter  in (7.19), we obtain

x  x0 y  y0 z  z0
(d ) :   (7.20) Canonical equations
u1 u2 u3

Note: The canonical, parametric and vector equations of the line (d ) passing through two

points P1 ( x1 , y1 , z1 ) and P2 ( x2 , y2 , z2 ) are respectively given by

 x   x1   x2  x1 
x  x1 y  y1 z  z1      
(d ) :   ; (d ) :  y    y1     y2  y1  ; (d ) : [r  r1 , r2  r1 ]  0
x  x1 y2  y1 z2  z1
 z   z1   z3  z1 

By Theoneste Hakizimana
78

Example: Write the equations of the straight line passing through the point P(2, 3, 4) and
x  2 y  3 z 1 x4 y0 z 4
perpendicular to the lines  d1  :   and  d 2  :  
1 1 1 2 1 3

Solution. Since the direction vectors of d1  and d 2  are v1  1 1 1 and v2   2 1 3


T T

then the direction vector of the line  d  perpendicular to both lines is v   v1 , v2    4 1 3


T

x2 y3 z 4
Consequently, the canonical equations of the line  d  are  d  :  
4 1 3

Distance from a point to a line

Pr..The distance from a point P0 with radius vector r0 to the line (d ) : r  r1  u is

 r1  r0 , u 
d  P0 , (d )   (7.21)
u

 x0   x1   u1 
     
In co-ordinates, if r0   y0  , r1   y1  and u  u2  , We have
 z0   z1   u3 

y1  y0 z1  z0 z z x1  x0
2 2
x x y1  y0
2

 1 0  1 0
u2 u3 u3 u1 u1 u2
d  P0 , (d )  
u12  u22  u32

x  2 y  3 z 1
Example.Find the distance from the point P(2, 3, 4) to the line  d  :  
1 1 1

Solution. The distance from the point P(2, 3, 4) to the line  d  passing through the

 P0 P, u 
point P0 (2,3, 1) with direction vector u  1 1 1 is given by d  P0 , (d )    
T
.
u

By Theoneste Hakizimana
79

Since P0 P   4 6 5 ;  P0 P, u    1 1 2 ; u  3;  P0 P, u   6; We have,
T

 P0 P, u 
 
d  P0 , (d )    2
u

3. Relative positions of two straight lines

Def. In 3D space two lines are said coplanar if they are in the same plane and they are called
skew lines if they are neither parallel nor intersecting. Two coplanar lines are either concurrent
or parallel or coincide.Let (d1 ) : r  r1  u and (d2 ) : r  r2  v be two lines. 4scenario occur:

(i) The lines (d1 ) and (d 2 ) are parallel if and only if u , v   0

(ii)The lines (d1 ) and (d 2 ) coincide if and only if u , v   0 and  r2  r1 , u   0

(iii) The lines (d1 ) and (d 2 ) are concurrent if and only if u , v   0 and r2  r1 , u , v  0

(iv) The lines (d1 ) and (d 2 ) are skew if and only if u , v   0 and (r2  r1 , u , v )  0

Pr.The distance between two parallel lines (d1 ) : r  r1  u and (d 2 ) : r  r2   u is given by

 r2  r1 , u 
d  (d1 ), (d 2 )   (7.22)
u

Pr.The distance between two skew lines (d1 ) : r  r1  u and (d2 ) : r  r2  v is given by

r2  r1 , u , v
d  (d1 ), (d 2 )   (7.23)
u , v 

x2 y4 z4 x  3 y 1 z  3


Example: Find out whether  d1  :   and  d 2  :   are skew
2 2 1 1 2 3
lines. If they are skew, find the distance between them.

Solution: In vector form, the equations of the lines d1  and d 2  may be written

By Theoneste Hakizimana
80

 d1  : r  r1   u ;  d 2  : r  r2   v , r1   2 4 4 ; u   2 2 1 ; v  3 1 3 ;
T T T

v   1 2 3 .The lines d1  and d 2  are skew if and only if the box product of
T

 
vectors r2  r1 , u1 and u 2 is different from zero. Since

r2  r1  1 5 1 ; r2  r , u , v  33  0 , then the two lines are skew. Since


T

4
u , v   7  ; u , v   101 , the distance d between d1  and d 2  is
 6 

r2  r1 , u , v 33
d 
u , v  101

4. Relative position between a straight line and a plane

Let consider the straight line  d  : x  x0  u1 ; y  y0  u2 ; z  z0  u3 and the plane

 : Ax  By  Cz  D  0

The relative position of the plane and the line depends on the solution set of the following system

 Ax  By  Cz  D  0

 x  x0  u1 , y  y0  u2 , z  z0  u3

Pr.The possible relative positions between the plane  and the line (d ) are

1. The line and the plane do not have common points i.e (d )    

This occurs if and only if Au1  Bu2  Cu3  0 and Ax0  By0  Cz0  D  0

2. The line intersects the plane at one point i.e (d )    P

(d ) and  have a single common point if and only if Au1  Bu2  Cu3  0

3. The line (d ) is contained in the plane  i.e (d )  

(d )   if and only if Au1  Bu2  Cu3  0 and Ax0  By0  Cz0  D  0

By Theoneste Hakizimana
81

The angle  between the line (d ) : r  r0  u and the plane  : r  r0 , N  0 is given by

N,u Au1  Bu2  Cu3


sin    (7.30)
N u A2  B 2  C 2  u12  u22  u32

The line (d ) is parallel to the plane  i.e (d )   N , u  0 and

The line (d ) is perpendicular to the plane  i.e (d )    u , N   0

Example. Find the angle  between  d  : x  2  3t ; y  1  t ; z  4t and  : x  2 y  z 1  0

Solution. The angle  between the direction vector u   3 1 4 of the line d  and the
T

 u, N 
normal vector N  1 2 1 to the plane   is given by sin  
1

T

u N 2 39

Example. Find the equation of the plane passing through the point P(4, 3,1) parallel to the
x y z x 1 y  3 z  4
straight lines  d1  :   ;  d2  :  
6 2 3 5 4 2

Solution. The direction vectors of the lines d1  and d 2  are u   6 2 3 and
T

v  5 4 2 respectively. Let M x, y, z  be an arbitrary point that belongs to the required


T

plane. PM   x  4, y  3, z  1 . Since the vectors PM , u and v are coplanar, then

x4 y3 z 1
PM , u , v  0 .In coordinates, we can write: 6 2 3  0
5 4 2

Expanding the determinant, the equation of the plane is  : 16x  27 y  14z  159  0

By Theoneste Hakizimana
82

Unit8: SECOND ORDER SURFACES IN STANDARD FORM

1.18. 8.1. SPHERE


8.1.1. Various equations

Def. A sphere is the set of all points in a space equidistant from a given point called the center.
The distance from the center to an arbitrary point of the sphere is called the radius.

A sphere (S ) of radius r with center at point C( ,  ,  ) is the set of all points P( x, y, z) in

space satisfying the condition d (C , P)  CP  r .Applying the distance formula, we have

r  ( x   )2  ( y   )2  ( z   )2 .

Squaring both sides of the equality we obtain:

S : ( x   )2  ( y   )2  ( z   )2  r 2 (8.1) : standard equation of the sphere

Expanding (8.1), we have

S : x2  y 2  z 2  2mx  2ny  2 pz  q  0 (8.2)

Where, m   , n   , p   and q   2   2   2  r 2

The equation of the sphere which passes through four points A( x1 , y1 , z1 ) , B ( x2 , y2 , z2 )

C ( x3 , y3 , z3 ) and D( x4 , y4 , z4 ) is given by

By Theoneste Hakizimana
83

x2  y 2  z 2 x y z 1
x12  y12  z12 x1 y1 z1 1
S : x2 2  y2 2  z2 2 x2 y2 z2 1 0 (8.3) : 4-point form equation
x32  y32  z32 x3 y3 z3 1
x4 2  y4 2  z4 2 x4 y4 z4 1

Example. Find the equation of the sphere which passes through the points
A(0,0,0), B(0,0,3), C(0, 2,0) and D(1, 2,1) .

Solution. Let the equation of the sphere be written in the form


S : x2  y 2  z 2  2mx  2ny  2 pz  q  0

Injecting the coordinates of each of the four points in succession into the equation of the
sphere, we obtain the following system of equations

q  0 ; 9  6 p  0 ; 4  4n  0 ; 2m 1  0

Solving, we get (m, n, p, q)  (0.5, 1, 1.5,0)

Therefore, the equation of the sphere which passes through the 4 points is

S : x2  y 2  z 2  x  2 y  3z  0

If we apply the 4-point form equation, we obtain the same answer.

2
3  7 
2 2
 1 
In canonical form it may be written S :  x     y  1   z    
2

 2  2   2 

1 3 7
The center and radius of this sphere are C  ,1,  and r  respectively.
2 2 2

The parametric equations of the sphere S of radius r with center C( ,  ,  ) are given by

 x    r cos u sin v

S :  y    r sin u sin v (8.4)
 z    r cos v

By Theoneste Hakizimana
84

Example: Find the equation of the sphere with its center at point C(3,6, 4) and tangent

to the plane  : 2 x  2 y  z  10  0 and write its parametric equations.

Solution: The radius of the sphere r is given by the distance from the point C to the plane 
3  2  2  6  4  10
We have: r  d (C ,  )  4
4  4 1

Hence, applying the formula (6.1) the required equation is

 x  3 2   y  6  2   z  4  2  16

Its parametric equations are

 x  3  4 cos u sin v

S :  y  6  4sin u sin v
 z  4  4 cos v

8.1.2. Intersection of a sphere and a line

To determine the points of intersection of the line

(d ) : x  x0  v1; y  y0  v2 ; z  z0  v3 (8.4)

With the sphere (8.2), substitute the values of x, y, z from (8.4) in (8.2) and arrange in powers of

 . The result is:

a 2  2b  c  0 (8.5)

Where, a  v12  v22  v32 , b   x0  m  v1   y0  n  v2   z0  p  v3


,

c  x02  y02  z02  2mx0  2 ny0  2 pz0  q

The roots in  of the quadratic equation (8.7) are the distances from the point M 0  x0 , y0 , z0  on

the line (8.6) to the points in which this line intersects the sphere.

Three cases may occur:

By Theoneste Hakizimana
85

a. The equation admits a single solution. The line is tangent to the sphere and the single
solution is the point of tangency.

b. The equation has no solution. The line passes out of the sphere and the line and the
sphere do not intersect.

c. The equation has two real distinct roots. The line meets the sphere in two points.

Example

Find the coordinates of the points of intersection of the of line

(d ) : x  1  ; y  1  ; z   and the sphere S : x2  y 2  z 2  5  0

Solution.

Substituting x  1  ; y  1  ; z   into the equation of

the sphere S : x2  y 2  z 2  5  0 , we get quadratic equation, 3 2  3  0 ,

Solving, we obtain two real distinct roots 1  1; 2  1 .

This means that the line intersects the sphere in two points:

P1 (0, 2, 1) If 1  1 and P2 (2, 0,1) If  2  1 .

Example

Find the coordinates of the points of intersection of the line

(d ) : 2 x  y  2 z 12  0; 2 x  4 y  z  6  0 and the sphere

S : x2  y 2  z 2  2x  2 y  4z  43  0

Solution

The parametric equations of the line (d ) may be written

(d ) : x  3; y  2; z  6  2

Substituting these values into the equation of the sphere

S : x2  y 2  z 2  2x  2 y  4z  43  0 , we get the quadratic equation,

By Theoneste Hakizimana
86

 2  2  1  0 ,

Solving, we obtain a double root 1  2  1 .This means that the line is tangent to

the sphere. Substituting the value of the root obtained into the equation of the line ,

we find that the tangency point is (3, 2, 4)

8.1.3. Plane section of a sphere

The relative position of a plane  : Ax  By  Cz  D  0 and a sphere

S : x2  y 2  z 2  2mx  2ny  2 pz  q  0

is obtained by solving the system of equations

 Ax  By  Cz  D  0
 2
 x  y  z  2mx  2ny 2 pz  q  0
2 2

Three cases may occur:

i.The system admits a single solution. In this case, the plane is tangent to the sphere and
the single solution is the point of tangency.

ii.The system has no solution. In this case, the plane passes out of the sphere and the plane
and the sphere do not intersect.

iii.The system has infinity of solutions. In this case, the intersection of this plane

with the sphere describes a circle.

Example: Find the equation of the section of the plane  : z  3  0 with the sphere
S : x 2  y 2  z 2  25

Solution. Substituting the equation of the plane into the equation of the sphere, we have

x2  y 2  9  25  x2  y 2  16 .The section obtained is a circle with radius 4 centered at the


point C (0,0,3) and lying in the plane  : z  3

By Theoneste Hakizimana
87

Example: Find the relative position of the plane  : x  5  0 and the sphere
S : x2  y 2  z 2  2x  4 y  2z  4  0

Solution

Substituting the equation of the plane into the equation of the sphere, we have

y 2  z 2  4 y  2 z  11  0 or  y  2    z  1  6
2 2

This means that the plane and the sphere do not intersect since the above equation represents
an empty set.

Tangent lines and tangent plane to the sphere

Pr.The equation of the tangent plane  to the sphere S : x2  y 2  z 2  2mx  2ny  2 pz  q  0

at the point M ( x0 , y0 , z0 ) is given by

 : x0 x  y0 y  z0 z  m( x0  x)  n( y0  y)  p( z0  z )  q  0 (8.6)

Example: Given the point M (3,1, 4) and the sphere S : x2  y 2  z 2  6 x  24 y  8z  0 . Prove


that the point M lies on the sphere and write the equation of the tangent plane at that point.

Solution

The given point lies on the sphere since its coordinates satisfy the equation of the sphere.

S (3,1, 4)  9  1  16 18  24  32  0 . Using the formula,

x0 x  y0 y  z0 z  m( x0  x)  n( y0  y)  p( z0  z )  q  0 ,

We have, 3x  y  4 z  3( x  3)  12( y  1)  4( z  4)  0

Therefore, the equation of the tangent plane is z  1

By Theoneste Hakizimana
88

8.2. CANONICAL EQUATIONS OF QUADRIC SURFACES

Def.2. A quadric surface or a conicoid is a surface defined by an equation of the second degree
in 3 variables. Any plane section of a SOS is a quadric curve or a limiting form of a
quadric curve.
1. Ellipsoid

Def. An ellipsoid is a quadric surface given by the equation

x2 y 2 z 2
   1; (a  0, b  0, c  0) (8.7)
a 2 b2 c2

The points (a,0,0);(0, b,0);(0,0, c) are called vertices of the ellipsoid.

The quantities a, b, c are called semi axes of the ellipsoid.

If a  b  c , the ellipsoid is transformed into a sphere whose equation is

x2  y 2  z 2  a2

The ellipsoid is a symmetric with respect to each coordinate plane: x  0; y  0 and z  0 .

It is also symmetric with respect to the origin of co-ordinates, which is its center.

By setting one of the variables at time equal to zero, we find the trace equations to be

x2 y 2 x2 z 2 y2 z2
  1,   1,   1 . These equations represent ellipses.
a 2 b2 a2 c2 b2 c2

The intersection of the ellipsoid and the plane z  h;  c  h  c is an ellipse defined by

x2 y 2 h2
  1 
a 2 b2 c2

By Theoneste Hakizimana
89

If two semi- axes are equal, the ellipsoid (8.7) is an ellipsoid of revolution generated by
x2 y 2 z 2 x2 y 2 z 2
revolving the ellipse around the third axis. We have:    1;   1
a2 a2 c2 a2 c2 c2

If the center of the ellipsoid is at P0 ( x0 , y0 , z0 ) and its axes are parallel to the coordinate axes,

the equation has the form:

 x  x0   y  y0   z  z0 
2 2 2

  1
a2 b2 c2 (8.8)

The parametric equations of the ellipsoid (8.7) may be written

 x  a cos u sin v

 y  b sin u sin v
 z  c cos v
 (8.9)

x2 y 2 z 2
Example. Find the intersection of the ellipsoid    1 and the plane  : x  2  0 .
16 12 4

x2 y 2 z 2 y2 z2
Solution. Solving the system,    1; x  2 , we obtain,  1
16 12 4 9 3

The section represents an ellipse whose vertices are (2, 3, 0);(2, 0,  3) .

2. Hyperboloids

a. Hyperboloid of one sheet

Def.The hyperboloid of one sheet is a quadric surface given by

x2 y 2 z 2
   1; (a  0, b  0, c  0) (8.10)
a 2 b2 c2

The hyperboloid of one sheet is symmetric with respect to each coordinate plane: x  0; y  0
and z  0 . It is also symmetric with respect to the origin of the coordinates, which is also its

By Theoneste Hakizimana
90

center. The points (a,0,0);(0, b,0) are vertices. The section made by the plan z  h is an

x2 y 2 h2
ellipse of equation 2  2  1  2 .
a b c

The sections made by the plane x  h and y  h are hyperbolas whose respective equations are

y2 z2 h2 x2 z 2 h2
  1  and   1 
b2 c 2 a2 a2 c2 b2

If h  a or h  b those hyperbolas are transformed into two lines

b a
y z or x   z respectively.
c c

The parametric equations of the hyperboloid of one sheet (8.10) may be written

a 1 b 1 c 1
x   u   cos v ; y   u   sin v ; z   u   (8.11)
2 u 2 u 2 u

x2 y 2 z 2
Example. Find the intersection of the hyperboloid of one sheet    1 and the plane
32 18 2
 : z 1  0 .

x2 y 2 z 2 x2 y 2
Solution. Solving the system,    1; z  1 , we obtain,  1
32 18 2 16 9

The section represents the hyperbola whose vertices are (4,0, 1);(4,0, 1) .

b. Hyperboloid of two sheets.

Def. The hyperboloid of two sheets is a quadric surface defined by the equation

By Theoneste Hakizimana
91

x2 y 2 z 2
   1;(a  0, b  0, c  0) (8.12)
a 2 b2 c 2

The hyperboloid of two sheets is symmetric about the planes x  0, y  0 and z  0 .

It is also symmetric with respect to the origin of the co-ordinates which is also its center.

y2 z2 x2
The equation (8.12) may also be written,   1 
b2 c2 a2

The section of (8.12) made by the plane x  h; h  a is an ellipse of equation

y2 z2 h2
  1 
b2 c2 a2

The sections of (8.12) made by a plane z  h or y  h are hyperbolas with respective equations

x2 y 2 h2 x2 z 2 h2
  1  or   1  . The points (  a, 0, 0) are the vertices of the surface (8.12)
a 2 b2 c2 a2 c2 c2

The parametric equations of the hyperboloid (8.17) may be written

a 1 b 1 c 1
x  u   cos v ; y   u   sin v ; z  u   (8.13)
2 u 2 u 2 u

By Theoneste Hakizimana
92

3. Paraboloid

a. Elliptic paraboloid

Def. The elliptic paraboloid is a quadric surface given by the equation

x2 y 2
  2 z;  p  0, q  0  (8.14)
p q

(8.14) is symmetric with respect to the planes of equations: x  0 of and y  0

The section of surface (8.14) made by the plane z  h; h  0 is an ellipse of equation:

x2 y 2
  2h
p q

The sections by the planes x  h and y  h are parabolas of equations:

 h2   h2 
y 2  2q  z   and x 2
 2 p  z   respectively.
 2p   2q 

The parametric equations of the elliptic paraboloid (8.14) may be written

u2
x  u p cos v ; y  u q sin v ; z  (8.15)
2

By Theoneste Hakizimana
93

b. Hyperbolic paraboloid

Def.The hyperbolic paraboloid is a quadratic surface given by the equation

x2 y 2
  2 z;  p  0, q  0  (8.16)
p q

The surface (8.16) is symmetric about the planes x  0 and y  0

x2 y 2
The section of (8.16) made by the plane z  h is a hyperbola of equation,   2h
p q

The sections by the planes x  h and y  h are parabolas of equations

x2 h2 y 2 h2
 2z  and   2 z respectively.
p q q p

The parametric equations of the hyperbolic paraboloid (8.16) may be written

x p (u  v) ; y  q (v  u ) ; z  2uv (8.17)

x2 y 2
Example. Find the intersection of the hyperbolic paraboloid   6 z and the plane
5 4
 : y6  0.

x2 y2  3
Solution. Solving the system,   6 z; y  6 , we get, x 2  30  z  
5 4  2

The section represents the equation of the parabola whose vertex is (0, 6, 1.5) .

By Theoneste Hakizimana
94

4. Quadric cone

Def. The quadric cone is a quadric surface defined by the equation

x2 y 2 z 2
   0; (a  0, b  0, c  0) (8.18)
a 2 b2 c2

The surface (8.18) is symmetric about the planes x  0, y  0 and z  0 . It is also symmetric with
respect to the origin of coordinates which is also its center.

x2 y 2 h2
The section of (8.18) made by a plane z  h is an ellipse of equation   .
a 2 b2 c2

The sections of (8.18) made by the planes x  h and y  h are hyperbolas of equations:

z 2 y 2 h2 z 2 x2 h2
  and   respectively. The section of (8.18) made by a plane y  hx
c2 b2 a 2 c2 a 2 b2

1 h2
consists of a couple of concurrent lines of equations, z  cx 
a 2 b2

1.19. 5. Cylindrical surfaces


Def8.A cylindrical surface is generated by straight line which moves a long fixed curve and
remains parallel to a fixed straight line. The fixed curve is called the directrix of the surface and
the moving line is the generatrix of the surface.

a. Elliptic cylinder

Def. An elliptic cylinder is a cylinder whose directrix is an ellipse.

The elliptic cylinder is described by the equation:

By Theoneste Hakizimana
95

x2 y 2
  1; (a  0, b  0) (8.19)
a 2 b2

If a  b , we have a circular cylinder whose equation is x 2  y 2  a 2

x2 y 2
The straight line parallel to the z  axis is the generatrix and the ellipse   1; z  0 is the
a 2 b2
directrix

b. Hyperbolic cylinder

Def. A hyperbolic cylinder is a cylinder whose directrix is hyperbola.

The hyperbolic cylinder is described by the equation:

x2 y 2
  1;(a  0, b  0) (8.20)
a 2 b2

The generatrix is a straight line parallel to the z  axis and the directrix is the hyperbola

x2 y 2
  1; z  0
a 2 b2

By Theoneste Hakizimana
96

c. Parabolic cylinder

Def. A parabolic cylinder is a cylinder whose directrix is a parabola.

The parabolic cylinder is described by the equation:

y 2  2 px; p  0 (8.21)

The generator of the parabolic cylinder is a line parallel to z  axis and the directrix is a
parabola, y 2  2 px; z  0

6. Planes
a. Real intersecting planes

A couple of real intersecting planes is described by the equation

a 2 x 2  b2 y 2  0 (8.22)

a
The directrices of the surface (8.22) are the lines of equation, y   ; z  0
b

b. Parallel planes

A couple of parallel plans is described by the equation:

x2  a2  0 ;(a  0) (8.23)

The equation (8.23) defines two lines x  a in the xy  plane

c. Coincident planes

A couple of coincident planes is described by the equation : z 2  0 (8.24)

By Theoneste Hakizimana
97

Example

By completing squares in x, y and z , determine the nature of the quadric surface whose
equation is:

a. 2 x2  3 y 2  z 2  8x  6 y  4 z  3  0

b. 3x2  4 y 2  2 z 2  6 x 16 y  8z  13

c. 3x2  2 y 2 12 z  6 x  8 y 13  0

Solution

a. By completing squares in x, y and z , we have:

2x 2  4 x  4  3y 2  2 y  1  z 2  4 z  4  13

2 x  2   3 y  1   z  2   18
2 2 2

Dividing the equation by 18, we obtain

 x  2  y  1  z  2
2 2 2

  1
 6  
2 2
32 3 2

The surface is an ellipsoid with the center at (2, 1, 2) and semi-axes

a  3, b  6, c  3 2

b. By completing squares in x, y and z , we have

3x 2  2 x  1  4y 2  4 y  4  2z 2  4 z  4  24

x  12   y  22  z  22 1


8 6 12

This is a hyperboloid of one sheet with center at the point (-1, 2,2) and axis parallel to
the z-axis.

By Theoneste Hakizimana
98

c. By completing the squares in x and y in x and y, we have:

3x 2  2 x  1  2y 2  4 y  4  12z  24

Regrouping, we have: 3 x  1  2 y  2   12 z  2 


2 2

Dividing both sides by 12, we get:


x  12   y  22 
z2
4 6 1

This is an elliptic paraboloid with the vertex at (1, 2, 2)

Example: In terms of the parameter k discuss on the type of the family of surfaces given
by: a. kx 2  y 2  z 2  1 b. x 2  ky 2  kz c. x2  y 2  k

Solution.

a. If k  0 , we have an ellipsoid

If k  0 , we have a circular cylinder

If k  0 , we have a hyperboloid of one sheet

b. If k  0 , we have an elliptic paraboloid

If k  0 , we have two parallel planes

If k  0 , we have a hyperbolic paraboloid

c. If k  0 , we have a hyperbolic cylinder

If k  0 , we have two concurrent planes

By Theoneste Hakizimana
99

Unit9: GENERAL EQUATION OF SECOND ORDER SURFACES


9.1. GENERAL EQUATION OF A QUADRIC SURFACE

 
In 3D-Affine Euclidean space   A3 ,V , , endowed with an orthonormal frame

F  0; e1 , e2 , e3  , the general equation of a quadric surface is governed by

S  x, y, z  : a11 x 2  a22 y 2  a33 z 2  2a12 xy  2a13 xz  2a23 yz 


 2a1 x  2a2 y  2a3 z  a0  0 (9.1)

3 3
in which the coefficients aij are not all zero i.e.  a
i 1 j 1
ij 0.

In terms of quadratic and linear forms, the equation (9.1) may be written
S  x, y, z  : q  x, y, z   2l  x, y, z   a0  0 ,Where,

q  x, y, z   a11 x 2  a22 y 2  a33 z 2  2a12 xy  2a13 xz  2a23 yz

l  x, y, z   a1 x  a2 y  a3 z

In matrix form, the equation (9.1) is written

S ( X ) : X T AX  2BX  C  0 or S  X  : Xˆ T AX
ˆ ˆ 0

 a11 a12 a13   x


   
Where, B   a1 a2 a 3  ; C  a0 ; A   a12 a22 a23  ; X   y  ;
 a13 a23 a33   z 

 a11 a12 a13 a1   x


   y
ˆA   a12 a22 a23 a2 
;X  
 a13 a23 a33 a3  z
   
 a1 a2 a3 a0  1 

By Theoneste Hakizimana
100

9.2. SIMPLIFICATION OF THE GENERAL EQUATION

Passing from orthonormal coordinate system F  0; e1 , e2 , e3  to the orthonormal coordinate

system F   0, e1, e2 , e3  by means of the orthogonal transformation X  PX  , the equation

(9.1) can be reduced to the form

S  x, y, z    1 x2  2 y2  3 z 2  2a1 x  2a2 y  2a3 z   a0  0


(9.2)

Where, 1 , 2 and 3 are the eigenvalues of the matrix A and P is the orthogonal matrix

diagonalizing A .The columns of the matrix P are made by orthonormalized eigenvectors


corresponding to the eigenvalues 1 , 2 and 3 .

Pr. By means of an orthogonal transformation and a translation the general equation of a


quadric surface (9.1), can be reduced to one of the following 5 forms:

I: 1 x 2  2 y 2  3 z 2  a0  0,  12 3  0 

II: 1 x 2  2 y 2  2a3 z  0,  12 a3  0 

III: 1 x 2  2 y 2  a0  0,  12  0 

IV: 1 x 2  2a2 y  0,  1a2  0 

V: 1 x 2  a0  0,  1  0 

By Theoneste Hakizimana
101

9.3. Classification of quadric surfaces

The classification of quadric surfaces can be done in terms of the coefficients of reduced forms.

I. 1 x 2  2 y 2  3 z 2  a0  0

a0 a0 a0
If a0  0 , divide by  a0 and put  a 2 ;  b 2 ;  c 2
1 2 3

The signs being so chosen that a, b, c are real.

This gives the following four types

x2 y 2 z 2
I1.    1; ellipsoid
a 2 b2 c2
x2 y 2 z 2
I 2 . 2  2  2  1; hyperboloid of one sheet
a b c
2 2
x y z2
I 3 . 2  2  2  1; hyperboloid of two sheets
a b c
2 2
x y z2
I 4 . 2  2  2  1; imaginary ellipsoid
a b c

If a0  0 ’ the reduced forms are

x2 y 2 z 2
I5 .    0; imaginary cone
a 2 b2 c2
x2 y 2 z 2
I 6 . 2  2  2  0; real cone
a b c

II : 1 x 2  2 y 2  2a3 z  0

According as 1 and 2 have the same or opposite signs, the equation takes the form

x2 y 2
II 2 .   2 z; elliptic paraboloid
a 2 b2
x2 y 2
II 2 . 2  2  2 z; hyperbolic paraboloid
a b

By Theoneste Hakizimana
102

III . 1 x 2  2 y 2  a0  0

If a0  0, this may be written in the form

x2 y 2
III1.   1; elliptic cylinder
a 2 b2
x2 y 2
III 2 .   1; imaginary elliptic cylinder
a 2 b2
x2 y 2
III 3 .   1; hyperboloid cylinder
a 2 b2

If a0  0, the equation takes the form

x2 y 2
III 4 .   0; pair intersecting imaginary planes
a 2 b2
x2 y2
III 5 .   0; pair of intersecting planes
a 2 b2

IV . 1 x 2  2a2 y  0

The equation of the surface takes the form

x 2  2 py; parabolic cylinder

V . 1 x 2  a0  0

If a0  0, according to the sign of a0 , the equation can be written

V1. x 2  a 2 ; two parallel planes


V2 . x 2  a 2 ; two imaginary parallel planes

If a0  0, we have

V3 . x 2  0 ; , one plane counted twice

By Theoneste Hakizimana
103

9.4.CENTER OF A QUADRIC SURFACE

Def. A point P ( x0 , y0 , z0 ) is called the center of the quadric surface (9.1), if the origin of

coordinates is placed at that point; the equation (9.1) loses its linear part.

The translation of the origin of coordinates to the point P ( x0 , y0 , z0 ) is described by

x  x  x0 ; y  y  y0 ; z  z  z0

Substituting into the equation (9.1), we have

S ( x, y, z ) : a11


 x2  2a12
 xy  a22
 y2  a33
 z 2  2a1 x  2a2 y   2a3 y   a0  0 (9.3)

Where,

  a11 ; a22
a11   a22 ; a33
  a33

  a12 ; a23
a12   a23 ; a13
  a13

a1  a11 x0  a12 y0  a13 z0  a1



a2  a12 x0  a22 y0  a23 z0  a2
 a  a x  a y  a z  a
 3 13 0 23 0 33 0 3

a0  S ( x0 , y0 , z0 )

From the definition of the center of a quadric surface, P ( x0 , y0 , z0 ) is a center of S if and only if

a1  a2  a3  0 .

This condition is equivalent to the system of equations

a11 x0  a12 y0  a13 z0  a1  0



a12 x0  a22 y0  a23 z0  a2  0
a x  a y  a z  a  0 (9.4)
 13 0 23 0 33 0 3

In terms of partial derivatives, (9.4) can also be written as

By Theoneste Hakizimana
104

1
 2 S x ( x0 , y0 , z0 )  0

1 S S S
 S y ( x0 , y0 , z0 )  0 , Where, S x  ; Sy  ; Sz 
2 x y z
1
 2 S z ( x0 , y0 , z0 )  0

The equations (9.4) imply that centers of the quadric surface (9.1) are the intersecting points of
three planes governed by equations

1 : a11 x  a12 y  a13 z  a1  0 ,  2 : a12 x  a22 y  a23 z  a2  0 and  3 : a13 x  a23 y  a33 z  a3  0

If det( A)  0, the system (9.4) has unique solution and the three planes (9.4) intersect at a single

point which is the unique center of the quadric surface. In this case the surface is said to be
central.

If det( A)  0, the system (9.4) is either consistent and have many solutions or is inconsistent and
has no solution. In this case the quadric surface is said to be non-central.

Example. In terms of the parameter k discuss on the type of the family of surfaces whose
equation is 1  2k  x 2  y 2  z 2  2kyz  1  0

Solution. In matrix form, the family of quadric surfaces 1  2k  x 2  y 2  z 2  2kyz  1  0 is


written
1  2k 0 0 0   x
 0 1 k 0   y 
x y z 1 
 0
0
k 1 0  z
  
 0 0 0 1  1 
The matrix A associated with the quadratic part of the surface is
1  2k 0 0
A   0 1  k  ; det( A)  1  2k 1  k 1  k 
 0 k 1 

(i) Central surfaces: det A  0

By Theoneste Hakizimana
105

 1
If k  1,1,  , then, det  A   0 and the surface has a unique center which is the origin (0, 0, 0)
 2

The surface can be written in the form

1  2k x 2   y  kz 2  1  k 2 z 2 1

We have 3 eigenvalues: 1  1  2k ,  2  1; 3  1  k 2

If k  1, we have two positive eigenvalues and one negative. The surface is a
hyperboloid of one sheet.
1
If  1  k  , the three eigenvalues are positive, and the surface is an ellipsoid.
2
If k  0 , the surface is a unit sphere whose equation is x  y  z  1
2 2 2

1
If  k  1 , two eigenvalues are positive, and one is negative. The surface is a
2
hyperboloid of one sheet.
If k  1 , two eigenvalues are negative and one positive. The surface is a hyperboloid of two
sheets.
(ii) Non central surface: det( A)  0

 1 
det  A   0  k  1, ,1
 2 
If k  1, the equation of the surface may be written 3 x 2   y  z   1 . It is an elliptic
2

cylinder
2
1  1  3
If k  , the equation of the surface can be written,  y  z   z 2  1. We have an
2  2  4
elliptic cylinder.

If k  1 , the equation of the surface may be written, 2 x 2   y  z   1. This is a


2

hyperbolic cylinder.

By Theoneste Hakizimana
106

Example: Reduce to canonical form the following quadric surface

S ( x, y, z) : 7 x2  6 y 2  5z 2  4 xy  4 yz  6 x  24 y  18z  30  0
Solution.

 7 2 0 3  x 
 2 6 2 12   y 
S  x, y , z  :  x y z 1    0
 0 2 5 9   z 
  
 3 12 9 30  1 
 7 2 0 
A   2 6 2 ; det( A)  162
 0 2 5 

The quadratic surface has a unique center. The center of the surface is found by solving

7 x  2 y  3

2 x  6 y  2 z  12
2 y  5 z  9

Solving, we find that the coordinates of the center are 1, 2, 1

The characteristic polynomial of A is

7   2 0
PA     2 6   2   3  18 2  99  162
0 2 5  

Factorizing, we get PA         3   6    9 

The spectrum of A is Sp  A   {3, 6,9}

Let find the eigenvectors associated with the eigenvalues 1  3, 2  6 and 3  9 respectively.

4 x  2 y  0

If 1  3 , we obtain the system 2 x  3 y  2 z  0
2 y  2 z  0

1
Solving and normalizing, we obtain: v1  1 2 2 ; e1  1 2 2
T T

By Theoneste Hakizimana
107

x  2 y  0

If 2  6 , we obtain the system: 2 x  2 z  0
2 y  z  0

1
Solving and normalizing, we obtain: v2   2 1 2 ; e2   2 1 2
T T

2 x  2 y  0

If 3  9 , we obtain the system: 2 x  3 y  2 z  0
2 y  4 z  0

1
Solving and normalizing, we obtain: v3   2 2 1 ; e3   2 2 1
3

x   1 2 2  1 
  1    
The transformation  y    2 1 2    2  leads to
3
 z   2 2 2   1

S  x, y, z   : 3x2  6 y2  9 z 2  6  0,

Dividing both sides of equation by 6, the standard form of the quadric surface is

x 2 y  2 y  2
S:   1
2 1 2
3

This equation represents an ellipsoid.

Example. Reduce to canonical form the following quadric surface

S ( x, y, z) : x2  5 y 2  z 2  2 xy  2 yz  6 xz  2x  6 y  2 z  0
Solution

1 1 3 1 x 
  
1 5 1 3  y 
S  x, y , z  :  x y z 1  0
3 1 1 1  z 
  
 1 3 1 0 1 

By Theoneste Hakizimana
108

1 1 3
A  1 5 1 ; det( A)  36
3 1 1

Since det( A)  36 , the quadric surface has unique center.

The center is found by solving

 x  y  3z  1

 x  5 y  z  3
3x  y  z  1

 1 2 2
Solving, we find that the coordinates of the center are   , , 
 3 3 3

The characteristic polynomial of A is given by

1  1 3
PA     1 5 1      3   6    2 
3 1 1 

Factorizing, we get, PA         3   6    2 

The spectrum of A is Sp  A   {3, 6, 2}

The eigenvectors associated with the eigenvalues 1  3, 2  6 and 3  2 are obtained by


solving the following homogeneous systems

2 x  y  3 z  0

If 1  3 , we obtain the system:  x  2 y  z  0
3 x  y  2 z  0

1
Solving and normalizing, we obtain: v1  1 1 1 ; e1  1 1 1
T T

5 x  y  3 z  0

If 2  6 , we obtain the system:  x  y  z  0
3 x  y  5 z  0

By Theoneste Hakizimana
109

1
Solving and normalizing, we obtain: v2  1 2 1 ; e2  1 2 1
T T

3x  y  3z  0

If 3  2 , we obtain the system:  x  7 y  z  0
3x  y  3z  0

1
Solving and normalizing, we obtain: v3  1 0 1 ; e3  1 0 1
T T

 1 1 1   1 
 
3 6 2  x  3 
x      
   1 2     2 
0   y 
The transformation  y    , yields
3 6 3
 z     z   
 1 1 1     2 
 3 2   3 
 6

S  x, y, z    3x2  6 y2  2 z 2  1

In standard form, this can be written

x 2 y  2 z  2
S:   1
1 1 1
3 6 2

The obtained equation represents a hyperboloid of one sheet

Example. Reduce to canonical form the following quadric surface

S ( x, y, z) : 2 x2  2 y 2  3z 2  4 xy  2 xz  2 yz  4x  6 y  2z  3  0
Solution

 2 2 1 2  x 
  
2 2 1 3  y 
S  x, y , z  :  x y z 1  0
1 1 3 1  z 
  
 2 3 1 3 1 

 2 2 1
A   2 2 1 ; det( A)  0
1 2 3

By Theoneste Hakizimana
110

Since det( A)  0 , we have a non-central quadric surface.

The characteristic polynomial and the spectrum of the matrix A are given respectively by

2 2 1
PA     2 2 1   3  7 2  10  PA         5    2 
1 1 3

Sp  A   {2,5, 0}

The Eigen vectors corresponding to the Eigen values 1  2; 2  5; 3  0 are obtained by


solving the homogeneous systems given by:

2 y  z  0

If 1  2 , we obtain the system:  2 x  z  0
x  y  z  0

1
Solving and normalizing, we obtain: v1  1 1 2 ; e1  1 1 2
T T

If 2  5 , we obtain the system

3x  2 y  z  0

2 x  3 y  z  0
x  y  2z  0

1
Solving and normalizing, we obtain: v2  1 1 1 ; e2  1 1 1
T T

2 x  2 y  z  0

If 3  0 , we obtain the system: 2 x  2 y  z  0
 x  y  3z  0

1
Solving and normalizing, we obtain: v3  1 1 0 ; e3  1 1 0
T T

Substituting

By Theoneste Hakizimana
111

 1 1 1 
 
6 3 2  x
x    
 y   1 1 1   
   6 y
3 2   
 z    z  
 2 1 
 6 0 
 3 

We get,

S  x, y, z   : 2 x2  5 y2  6 x  5 2 z  3  0


2
 6  9 
S  x, y, z   : 2  x    5 y  5 2  z 
2
0
 4   20 2 

6 9
Setting, x  x  ; y  y; z  z  ,we obtain,
4 20 2

S  x, y, z  : 2 x2  5 y2  5 2 z  0,

Dividing both sides by 10, the canonical equation of the surface is written,

x2 y2 2
S  x, y, z  :   z .
5 2 2

This equation represents an elliptic paraboloid.

By Theoneste Hakizimana
112

By Theoneste Hakizimana

You might also like

pFad - Phonifier reborn

Pfad - The Proxy pFad of © 2024 Garber Painting. All rights reserved.

Note: This service is not intended for secure transactions such as banking, social media, email, or purchasing. Use at your own risk. We assume no liability whatsoever for broken pages.


Alternative Proxies:

Alternative Proxy

pFad Proxy

pFad v3 Proxy

pFad v4 Proxy